Pharmacology Final Exam

Pataasin ang iyong marka sa homework at exams ngayon gamit ang Quizwiz!

C (the image shows oral candidiasis, which is a potential consequence of oral/inhaled steroid treatment, due to immunosuppression - beclomethasone is an inhaled steroid indicated for the treatment of chronic asthma)

A 12-year-old female presents to her pediatrician with an asthma exacerbation. Upon physical examination, the physician notes the finding shown. This presentation is a common side effect of which of the following pharmacologic agents used in the treatment of asthma? A. Salmeterol B. Albuterol C. Inhaled beclomethasone D. Ipratropium E. Theophylline

B (the key is knowing which of these drugs is long acting - A, C, and E are short acting benzos, and D is a benzo antidote) (by the way the diagnosis was tetanus)

A 15-year-old male sustained a laceration to his left lower extremity during a football game and was stitched up by the EMT later that day. Six days later, the teenager began to experience neck rigidity, laryngospasms, dysphagia, and risus sardonicus. He was taken to the hospital where he was given treatment for an infection caused by a gram-positive, anaerobic bacili that consisted of metronidazole, immunoglobulins targeting the toxoin, and a long-acting medication that facilitates the GABA-A receptor by increasing the frequency of the chloride channel opening. Which of the following medications was he given? A. Triazolam B. Diazepam C. Oxazepam D. Flumazenil E. Midazolam

A ("methacholine challenge" should be synonymous with "asthma diagnosis" - methacholine is a direct cholinergic agonist that can induce an asthma attack in an asymptomatic agent, and is rapidly metabolized so is very short-acting) (B, D; these are direct cholinergic agonists used for glaucoma) (C; this is a direct cholinergic agonist used for urinary retention) (E; this is used for glaucoma and atropine poisoning)

A 16 year-old female is being evaluated for shortness of breath. For the last year she has had shortness of breath and subjective wheezing with exercise and intermittent coughing at night. She reports waking up from sleep coughing 1-2 times per month. She now skips gym class because of her symptoms. She denies any coughing, chest tightness, or shortness of breath on the day of her visit. On exam, her lungs are clear to auscultation bilaterally, with normal inspiratory to expiratory duration ratio. Her pulmonary function tests (PFTs) show normal FEV1 and FVC based on her age, gender, and height. She is told to inhale a medication, and her PFTs are repeated, now showing a FEV1 79% of her previous reading. The patient is diagnosed with asthma. Which of the following medications was used to diagnose the patient? A. Methacholine B. Pilocarpine C. Bethanechol D. Carbachol E. Physostigmine

E (the patient has status epilepticus - the inability to regain consciousness across multiple repeated and extended seizure episodes - the first line treatment for this is lorazepam)

A 16-year-old boy with history of seizure disorder is rushed to the Emergency Department with multiple generalized tonic-clonic seizures that have spanned more than 30 minutes in duration. He has not regained consciousness between these episodes. In addition to taking measures to ensure that he maintains adequate respiration, which of the following is appropriate for initial pharmacological therapy? A. Phenytoin B. Carbamazepine C. Gabapentin D. Valproic acid E. Lorazepam

C (loratidine AKA claritin is a second generation antihistamine [second generation = doesn't penetrate the CNS, doesn't cause drowsiness] that is indicated for allergy relief)

A 17-year-old male presents to his primary care physician with the complaint of rhinitis and watery eyes, especially in the spring. He states that he used to take medicine for this, but he stopped because it made him too drowsy. In order to treat his symptoms while decreasing the risk of drowsiness, the most appropriate medication for him would be A. Cyproheptadine B. Diphenhydramine C. Loratadine D. Scopolamine E. Zafirlukast

A (somnolence, miosis, and respiratory depression are symptoms of opioid use - the patient was likely given opioids for his pain - opioids work by depolarizing the neuron with potassium efflux)

A 17-year-old male presents with altered mental status. He was recently admitted to the hospital due to a tibial fracture suffered while playing soccer. His nurse states that he is difficult to arouse. His temperature is 98.6 deg F, blood pressure of 130/80 mm Hg, pulse of 60/min, and respirations of 6. Exam is notable for pinpoint pupils and significant lethargy. Which of the following describes the mechanism of action of the drug likely causing this patient's altered mental status? A. Neuronal hyperpolarization due to potassium efflux B. Neuronal depolarization due to potassium influx C. Neuronal hyperpolarization due to sodium influx D. Neuronal depolarization due to sodium efflux E. Neuronal hyperpolarization due to chloride influx

B (the patient is treating their bipolar disorder with lithium - a unique side effect of lithium therapy is hypothyroidism, as reflected in the clinical and laboratory findings)

A 19-year-old African American male with a history of bipolar I disorder presents to the psychiatrist for a follow-up visit. During the session, the patient explains that for the past 2 months he has felt significantly fatigued and constipated. He is always complaining of feeling cold and has gained several pounds although his diet has not changed. A blood sample was sent for analysis, revealing the following: TSH - 6 mIU/L (nl = 0.4-4.0 mIU/L), free T4 - 0.4 ng/dL (nl = 0.7-1.9 ng/dL), and serum T4 - 2.1 mcg/dL (nl = 4.6-12 mcg/dL). Which of the following is responsible for these abnormalities? A. Valproic acid B. Lithium C. Carbamazepine D. Lamotrigine E. Olanzapine

C (phenytoin is dramatically teratogenic and causes fetal hydantoin syndrome, which appears as described - A and B are not notably teratogenic, D causes cleft deformities, and E causes neural tube defects)

A 30-year-old G1P1 Caucasian female gives birth to an infant after having received no prenatal care. Upon physical exam, the pediatrician notices pronounced growth restriction with microcephaly in the neonate and several craniofacial abnormalities, hypoplastic growth of nails, decreased cranial motion, and apparent dysfunction with tongue thrusting. Intellectual disabilities are also suspected. The patient states she does take an antiseizure medication but is unsure of the name. Which medication below could have led to the congenital defects of the infant? A. Ethosuximide B. Gabapentin C. Phenytoin D. Topiramate E. Valproic acid

B (oral contraceptives dramatically decrease the risk of ovarian and endometrial cancer, but are associated with a slight increase in the risk of breast cancer and possibly in the risk of cervical cancer)

A 19-year-old female presents to your office for a refill on her combined oral contraceptive pills. She has no previous medical history and takes no other medication. She does report that a classmate told her that these pills increase the risk of ancer and asks you if this is true. You reply that combined oral contraceptives A. decrease the risk of breast cancer, decrease the risk of endometrial cancer, and decrease the risk of ovarian cancer B. increase the risk of breast cancer, decrease the risk of endometrial cancer, and decrease the risk of ovarian cancer C. increase the risk of breast cancer, decrease the risk of endometrial cancer, and do not affect the risk of ovarian cancer D. increase the risk of breast cancer, decrease the risk of endometrial cancer, and increase the risk of ovarian cancer E. increase the risk of breast cancer, do not affect the risk of endometrial cancer, and do not affect the risk of ovarian cancer

A (the tricyclic antidepressant overdose can be remembered as the three Cs; convulsions, cardiotoxicity, and antiCholinergic activity - this toxicity is why TCAs are not first choice for depression treatment - the widened QRS is an especially important feature of TCA toxicity) (B; this is a calcium channel blocker - toxicity would include hpotension, peripheral edema, dizziness, constipation, and a possible AV block of the heart) (C, D, E; these are all SSRIs, which all have the same toxicity of serotonin syndrome - hyperthermia, GI distress, myoclonus, and flushing - there would be no EKG changes)

A 20-year-old female presents to the emergency department with an altered mental status and convulsions. her roommate states that she took an unknown amount of an unknown medication in an effort to commit suicide. Physical examination reveals tachycardia with a heart rate of 128. Further vitals include a systolic blood pressure of 85 mmHg, respiratory rate of 22, temperature of 96.8, and pulse oximetry of 99% with use of a non-rebreather mask. Neurologic examination reveals large pupils with slow reactivity. Gag reflex is minimal, and there is no response to auditory stimuli with multiple repetitions of the patient's name. She does withdraw from pain in her extremities, and you are able to elicit 2+ reflexes bilaterally. Cranial rhythmic impulse is depressed, and there are no abnormal soft-tissue findings in muscle or fascia palpated. EKG shows a widened QRS. Of the following, the most likely medication causing this toxicity is A. Amitrytypline B. Amlodipine C. Fluoxetine D. Paroxetine E. Sertraline

D (infliximab is a TNF-alpha inhibitor, indicated for long term treatment of chronic inflammatory diseases like Crohn's - side effects include profound immunosuppresion, creating the likelihood of infections by fungal agents - this particular agent is likely histoplasmosis, given the patient's Midwestern region of origin) (A; is given for cancer, side effects are myelosuppresion and GI discomfort) (B; is an antibiotic, side effects are rashes, headache, dizziness, cartilage damage, and tendonitis) (C; appropriate for short term treatment of flare ups in Crohn's, but not appropriate long term due to severe metabolic side effects) (E; given for cancer, side effects include myelosuppression, fatty liver changes, and neural tube defects in fetuses)

A 21-year-old Midwestern Asian male is seen by his gastroenterologist for an acute flare-up episode of his Crohn disease. He has been managed on sulfasalazine but it has not been efficacious. The physician recommends a medication that is used for long-term treatment of Crohn's disease. Two weeks later he visits his primary care physician with a cough, malaise, and chest pain. Sputum culture reveals fungi-filled macrophages. The most likely pharmacological agent responsible for these symptoms was A. 6-mercaptopurine B. Ciprofloxacin C. Glucocorticoids D. Infliximab E. Methotrexate

D (third generation cephalosporin, a beta lactam that inhibits cell wall synthesis, being prescribed in this case to treat a disseminated gonnorhea infection) (A; this inhibits the 50S ribosomal subunit) (B; aminoglycoside, this inhibits the 30S ribosomal subunit) (C; fluoroquinolone, inhibits DNA gyrase) (E; sulfa drug, inhibits the synthesis of tetrahydrofolate)

A 21-year-old sexually active male complains of fever, pain during urination, and inflammation and pain in the right knee. A culture of the joint fluid shows a bacteria that does not ferment maltose and has no polysaccharide capsule. The physician orders antibiotic therapy for the patient. The mechanism of action of action of the medication given blocks cell wall synthesis, which of the following was given? A. Chloramphenicol B. Gentamicin C. Ciprofloxacin D. Ceftriaxone E. Trimethoprim

C (the patient is being treated for depression and insomnia, which is an indication for trazodone, a potential side effect of which is priapism)

A 23-year-old Caucasian male presents to the emergency department with a persistent penile erection for the last 6 hours. He recently began outpatient treatment for depression with associated insomnia. He traveled to Mexico 5 months ago. His medical history is otherwise unremarkable. Which of the following is the most likely precipitating factor for priapism in this patient? A. Depression treatment with bupropion B. Depression treatment with venlafaxine C. Depression treatment with trazodone D. Sickle cell disease E. Infection acquired in Mexico

A (you can remember the mechanism of the alpha1, alpha 2, beta1, and beta2 receptors with the mnemonic QISS, which tells you which G protein subtype is activated by those 4 receptors respectively - since Gs is stimulatory, it must increase cAMP)

A 23-year-old woman presents to the emergency department with acute onset of shortness of breath, wheezing, and chest tightness. This is her 4th visit for these symptoms in the last 5 years. She tells you she recently ran out of her normal "controller" medication. Concerned for an asthma exacerbation, you begin therapy with a short-acting beta2-agonist. What is the expected cellular response to your therapy? A. Gs protein coupled receptor activates adenylyl cyclase and increases intracellular cAMP B. Gs protein coupled receptor activates phospholipase C and increases intracellular calcium C. Gq protein coupled receptor activates phospholipase C and increases intracellular calcium D. Gq protein coupled receptor activates adenylyl cyclase and increases intracellular cAMP E. Gi protein coupled receptor inhibits adenylyl cyclase and decreases cAMP

E (vinblastine and vincristine are both alkaloid drugs that inhibit the formation of the mitotic spindle in dividing cells by binding to tubulin and inhibiting microtubule synthesis, arresting the cell in metaphase - side effects of these drugs include neuropathies and bone marrow suppression)

A 24-year-ld female with a past medical history of Hodgkin's lymphoma is started on a new antineoplastic medication. The medication binds tubulin to inhibit microtubule formation. The most likely drug the patient was started on is A. Bleomycin B. Cyclophosphamide C. Doxorubicin D. Methotrexate E. Vinblastine

A (any time a woman has cervicitis most likely caused by an STD, immediately suspect the two most frequent causes, Chlamydia and Gonorrhea - the treatment for chlamydia is either azithromycin or doxycycline [the former is preferred because it's only a single dose] and the treatment for gonorrhea is ceftriaxone)

A 25-year-old Caucasian female with a history of abnormal vaginal bleeding presents to her primary care physician. She has experienced spotting midcycle and end-of-cycle for the past two months with a gray discharge of mild odor. The patient is a graduate student at the local university and has a history of on new sexual partner for the past 6 months. A pelvic examination reveals cervical tenderness but a negative chandelier sign. Visualization of the cervix revealed erythema and a gray-white discharge surrounding the os. Swabs were taken for culture and sensitivity analysis. The most appropriate pharmacologic coverage for this patient's condition includes A. Azithromycin + ceftriaxone B. Ceftriaxone C. Ciprofloxacin D. Clarithromycin + metronidazole E. Doxycyline + erythromycin

E (the patient has hyperthyroidism - propranolol is a nonselective beta blocker that reduces cardiac symptoms like increased heart rate, and also reduces the peripheral conversion of T4 to T3)

A 25-year-old female presents to her primary care physician with complaint of tachycardia, frequent sweating, tremor, and she reports of "feeling hot". Laboratory results show decreased thyroid stimulating hormone. Of the following, the drug of choice to initially control her cardiac symptoms is A. Amiodarone B. Digoxin C. Methimazole D. Procainamide E. Propranolol

D (clomiphene is a partial estrogen agonist used to induce ovulation in anovulatory patients - common side effects are enlarged ovaries [due to overstimulation of multiple follicles], hot flashes, vasomotor flushing, multiple gestations, and visual disturbances)

A 26-year-old female comes to your clinic concerned about why she only has one period every four to five months on average. After stating she has been trying to conceive, you do a complete infertility work up and rule out all etiologies that might cause oligomenorrhea and/or amenorrhea. You conclude that she has an anovulatory dysfunction secondary to polycystic ovarian syndrome and decide to treat her with clomiphene. Of the following, the most likely side effect of taking clomiphene is A. Deep vein thrombosis B. Endometrial cancer C. Gynecomastia D. Enlarged ovaries E. Uterine contractions

A (the patient has signs of hepatocellular injury due to acetaminophen overdose, likely as a suicide attempt - alcoholism is a risk factor for this type of liver injury - the treatment is N-acetylcysteine, a glutathione substitute)

A 26-year-old man with a history of alcoholism presents to the emergency department with nausea, vomiting, and right upper quadrant pain. Serum studies show AST and ALT levels >5000 U/L. A suicide note is found in the patient's pocket. The most appropriate initial treatment for this patient has which of the following mechanisms of action? A. Glutathione substitute B. Heavy metal chelator C. Opioid receptor antagonist D. GABA receptor competitive antagonist E. Competitive inhibitor of alcohol dehydrogenase

E (the patient has renal artery bruits, which may indicate stenosis - in a young woman, that could mean fibromuscular dysplasia - but do not give ACE inhibitors like lisinopril to patients with decreased GFR, since those drugs further lower GFR and can cause renal hypoperfusion injury)

A 27-year-old female with no significant past medical history presents to clinic for an annual evaluation. She has no complaints. Vital signs are as follows: T 37 Celsius, HR 70, BP 159/93, RR 12, and O2 99% on room air. Her BMI is 20.1. Repeat blood pressures on two separate occasions are 157/95 and 161/91. On physical examination, you auscultate just laterally to the umbilicus and detect a low pitched "wooshing" sound on each side. In medical management of this patient's hypertension, which medication should be avoided? A. Hydrochlorothiazide B. Atenolol C. Amlodipine D. Spironolactone E. Lisinopril

E (SSRIs are first line drugs of choice for the treatment of panic disorder due to efficacy, lack of abuse potential, action against comorbid conditions, and lack of severe side effects)

A 27-year-old male is referred to a psychiatrist for episodes of severe anxiety. History reveals that the episodes are triggered by riding on trains, sitting in crowded movie theaters, and standing in lines and may be accompanied by chest pain, palpitations, and difficulty breathing. These symptoms resolve when he steps away to be alone. Which of the following is the most appropriate agent to prevent future episodes? A. Diazepam B. Haloperidol C. Lithium carbonate D. Lorazepam E. Paroxetine

D (SSRIs, which are first line for depression, do not act immediately, or even quickly - they require at least a month [4-6 weeks to be safe] to achieve therapeutic effect)

A 28-year-old female comes to clinic complaining of insomnia and decreased appetite. After interviewing the patient, the physician notes that the patient has been experiencing symptoms of anhedonia and guilt. The patient complains that her energy level and concentration are "not what they used to be." Thyroid function tests are normal. The physician decides to start the patient on a medication. What length of time must be given in order to allow the medication to provide symptomatic relief? A. 4-6 minutes B. 4-6 hours C. 4-6 days D. 4-6 weeks E. 4-6 months

D (the girl likely overdosed on diphenhydramine, which has antihistamine and anticholinergic effects - physostigmine is a cholinisterase inhibitor and can counteract the anticholinergic effects of diphenhydramine) (A; this is the antitode for organophosphate poisoning) (B; this is the antidote for acetaminophen overdose) (C; this is the antidote for opioid overdose) (E; this is the antidote for iron poisoning)

A 3-year-old girl swallowed a handful of pills after her grandmother dropped the bottle on the ground this afternoon. She presents to the ER in a very drowsy but agitated state. She is clutching her abdomen, as if in pain, her skin is dry and flushed, and she does not know her name or where she is. Her pupils are dilated. Her grandmother reports that she has not urinated in several hours. The grandmother's medical history is significant for allergic rhinitis and osteoarthritis, both of which are treated with over the counter medications. What is the appropriate treatment for this child? A. Atropine B. N-acetylcysteine C. Naloxone D. Physostigmine E. Deferoxamine

C (methimazole inhibits the synthesis of thyroid hormone by interfering with PTO, thyroid peroxidase, to trat hyperthyroidism)

A 30-year-old female complains of heat intolerance, excessive sweating, and recent unintentional weight loss. She denies alcohol, tobacco, and other drug use and does not take any medications. Laboratory analysis reveals elevated serum T4 and decreased serum TSH. You elect to manage the patient's condition pharmacologically with methimazole. Methimazole interferes with the function of which of the following proteins? A. Thyroxine-binding globulin B. Sex-hormone binding globulin C. Thyroid peroxidase D. 5'-deiodinase E. Sodium/iodide symporter

D (I dunno about you, but when I see a sweaty anxious farmer in the emergency room, I think organophosphate poisoning, which causes inhibition of AChE, overstimulating the nicotinic and muscarinic receptors with too much acetylcholine - the antidote is atropine to compete with the acetylcholine, and pralidoxime to reverse the AChE inhibition)

A 32-year-old farmer is brought to the emergency department by his wife. The patient was reportedly anxious, sweaty, and complaining of a headache and chest tightness before losing consciousness on route to the hospital. Which of the following is mechanistically responsible for this patient's symptoms? A. Competitive inhibition of acetylcholine at post-junctional effector sites B. Binding of acetylcholine agonists to post-junctional receptors C. Inhibition of presynaptic exocytosis of acethylcholine vesicles D. Irreversible inhibition of acetylcholinesterase E. Reversible inhibition of acetylcholinesterase

B (this is a tricyclic antidepressant, which increases the QT interval and predisposes to torsades de pointes, as shown in the EKG)

A 32-year-old female with a history of depression presents to the emergency department after a suspected ingestion. She is confused, reporting blurry vision, and responding to visual hallucinations. Vital signs are as follows: Temperature: 98.9 degrees Farenheit (37.2 Celsius) Heart Rate: 105 bpm Blood Pressure: 90/65 mmHg Respiratory Rate: 21 respirations per minute O2 Saturation: 99% on room air Upon reviewing her EKG, the emergency room physician orders sodium bicarbonate. What medication was the likely cause of this patient's cardiac abnormality? A. Lithium B. Amitryptiline C. Paroxetine D. Quetiapine E. Sertraline

C (methyldopa is an alpha 2 agonist commonly prescribed to treat hypertension in pregnant patients - common side effects are sedation and drowsiness - another drug that would work well in this situation is hydralazine) (the other drugs listed are either listed as pregnancy category C or are shown to have adverse effects on a developing fetus)

A 33-year-old female with benign essential hypertension, controlled with hydrochlorothiazide, was recently discovered to be pregnant. Her physician decides to substitute hydrochlorothiazide with another antihypertensive agent. This new agent is most likely A. Amlodipine B. Lisinopril C. Methyldopa D. Metoprolol E. Valsartan

B (the condition is myasthenia gravis, an autoimmune disorder in which antibodies attack the acetylcholine receptors at the neuromuscular junction - the treatment is neostigmine or pyridostigmine, to inhibit cholinesterase to increase the availability of acetylcholine in the synapse) (A; this describes the pathology of the condition, not the treatment) (C; this describes the action of corticosteroids, which is a good treatment for multiple conditions, but the extensive side effect profile makes the stigmine drugs a better choice) (D; this is the mechanism of NSAIDs, which would not be indicated here) (E; this describes pralidoxime, a muscarinic blocker to treat organophosphate poisoning)

A 34-year-old female presents to your office complaining of progressive muscle weakness of four days duration. her history reveals she is a night security guard and she has difficulty watching the security monitors, especially towards the end of her shift. She denies fever, recent illnesses, and history of trauma. Given her most likely disorder, what is the mechanism of action for the best pharmacologic therapy? A. antibodies competitively antagonizing the post-synaptic acetylcholine receptor B. blocking acetylcholinesterase, thereby increasing the acetylcholine to post-synaptic receptor ratio C. decreased production of prostaglandins through inhibition of phospholipase A2 D. decreased production of prostaglandins though inhibition of the cyclooxygenase enzyme E. reactivation of acetylcholinesterase at the neuromuscular junction

C (the stem is that of pheochromocytoma, a catecholamine secreting tumor of the adrenal gland that causes excess sympathetic stimulation - phenoxybenzamine blocks alpha 1 and alpha 2 receptors irreversibly, effectively controlling the symptoms of the disease - side effects include reflex tachycardia and orthostatic hypotension)

A 35-year-old female presents to her primary care physician with the complaint of headaches, tachycardia, and diaphoresis, which she reports have occurred intermittently over the past 3-4 months. Her blood pressure is 165/95 mmHg. A 24-hour urine collection study reveals the presence of vanillylmandelic acid. You note paravertebral hypertonicity from T10-11 and a tender nodule 2 cm superior and 1 cm to the left of the umbilicus. The drug therapy of choice includes A. Aspirin B. Ondansetron C. Phenoxybenzamine D. Propranolol E. Tamsulosin

E (this is the only drug of the given choices which is an NRTI) (A; this is an antiviral for herpes that inhibits DNA polymerase, and is not part of HAART) (B; this is a non-nucleoside reverse transcriptase inhibitor) (C, D; these are protease inhibitors)

A 35-year-old male presents for HIV monitoring. He has a sore throat but denies any other symptoms. His CD4 count is 198/uL and he is referred to the HIV clinic to begin Highly Active Antiretroviral Therapy (HAART), which is to include a nucleoside reverse transcriptase inhibitor (NRTI). Which of the following drugs would most likely be prescribed? A. Acyclovir B. Efavirenz C. Nelfinavir D. Saquinavir E. Zidovudine

D (infliximab is a monoclonal antibody for TNF alpha, in this case for a patient's rheumatoid arthritis - the only answer choice that is also a monoclonal antibody is Trastuzumab, which attacks Her2 positive breast cancer) (remember that the 'mab' suffix literally stands for 'monoclonal antibody')

A 36-year-old female presents with a 6-month history of stiffness in her joints. She reports bilateral knee pain that is worst in the morning prior to activity and stiffness of the fingers in both hands. Anti-CCP antibody tests are positive. Over several months, the patient's symptoms prove unresponsive to NSAIDs and methotrexate, and the decision is made to begin infliximab. Which of the following is in the same therapeutic drug class as inflixmab? A. Imatinib B. Indomethacin C. Cyclophosphamide D. Trastuzumab E. Allopurinol

E (this patient likely has organophosphate poisoning, the symptoms of which are hypercholinergic [hypersecretory], for which the treatment is atropine, an anticholinergic)

A 37-year-old farmer presents to the emergency department with acute onset of complaints of diarrhea, excessive tearing, and increased saliva production. He is concerned that he is dehydrated, as he has also been urinating with increased frequency over the past several hours. Physical exam is significant for a moderately agitated, diaphoretic male who demonstrates pinpoint pupils. You suspect an exposure to a toxic substance might be the cause of his symptoms. Which of the following pharmacologic treatments would be most appropriate for the likely exposure? A. Neostigmine B. Bethanechol C. Donepezil D. Pilocarpine E. Atropine

E (the patient is experiencing Prinzmetal angina, the treatment for which is a calcium channel blocker or a nitrate to prevent coronary vasoconstriction - diltiazem is the former kind of drug) (A, B; these are great treatments for hypertension, but not for angina) (C, D; these would actually make the angina worse as they could make vasoconstriction worse)

A 37-year-old female presents to the general medical clinic with chest pain. She reports that the pain occurs at rest and feels as if an elephant is sitting on her chest. She has no other complaints. Her past medical history is significant for migraines but she has no history of smoking, hypertension, hyperlipidemia, or diabetes. Her vital signs are temperature 37 degrees Celsius, HR 70/minute, BP 110/80, RR 16/min, and oxygen saturation 99% on room air. Her physical examination reveals no murmurs. An EKG in the office during one of the episodes reveals the following as shown. Troponins are positive. She is admitted to hospital and undergoes emergent cardiac catheterization, where she is without obstructive coronary disease, but her symptoms can be provoked with administration of intravenous ergonovine. Which of the following treatments would be appropriate in this patient? A. Furosemide B. Lisinopril C. Metoprolol D. Aspirin E. Diltiazem

D (omeprazole is the prototype PPI that irreversibly inhibits the H/K ATPase in stomach parietal cells - this causes a decrease in acid secretion which can assist in H pylori treatment by reducing the exacerbation or pain of ulcers, while the antibiotics attack the microbes) (A; this is the mechanism of octreotide, a somatostatin analogue) (B; this is the mechanism of muscarinic antagonists) (C; this is the mechanism of cimetidine, ranitidine, famotidine, and nizatidine) (E; this is the mechanism of some antibiotics, like macrolides)

A 37-year-old woman presents to your office with heartburn. Following an extensive workup, you diagnose her with a duodenal ulcer secondary to H. pylori infection. You prescribe triple therapy of omeprazole, clarithromycin, and amoxicillin. Which of the following best describes the mechanism of action of omeprazole? A. ST2 receptor antagonism B. Parasympathetic inhibition C. Reversible block of H2 receptors D. Inhibition of primary active transport E. Inhibition of 50S ribosomal subunit

E (praziquantel is an antihelminthic agent indicated for treatment of liver flukes like Clonorchis sinensis, as well as other trematodes and cestodes - the drug increases the cell membrane permeability of the parasite, leading to disintergration) (A; this is an antihelminth indicated for filiriases caused by roundworms) (B; this is a drug indicated for a variety of parasites and anaerobes) (C; this drug is used to treat Chagas disease) (D; this drug is used to treat schistosomiasis and dracunculiasis)

A 38-year-old male presents to the clinic for evaluation of recent onset abdominal pain. He recently emigrated from Southeast Asia to the United States. Stool studies isolate eggs identified as Clonorchis sinensis The most appropriate pharmacologic treatment is A. Diethylcarbamazine B. Metronidazole C. Nifurtimox D. Niridazole E. Praziquantel

A (exenatide is a GLP1 receptor agonist that acts to enhance insulin secretion - this controls fasting and posprandial glucose concentration - side effects include nausea, vomiting, and acute pancreatitis, as in the stem) (B; this is pramlintide) (C; this the sulfonlyureas, like glimepiride) (D; this miglitol and acarbose) (E; this is the gliptins, like sitagliptin)

A 38-year-old male with uncontrolled diabetes mellitus type 2 has recently started exenatide in order to better control his glucose levels. Five days after beginning this new regimen, he presents to the emergency department with intractable mid epigastric abdominal pain that radiates to his back. A Chapman point is located anteriorly on the right, in the cartilage between the 7th and 8th ribs. Laboratory values reveal a blood glucose level of 60 mg/dL, along with increased amylase and lipase levels. Exenatide exerts its effects through A. Agonism of glucagon-like-peptide-1 receptors B. Amylin peptide analogue C. Closure of potassium gated channels D. Inhibition of alpha-glucosidase E. Inhibition of dipeptidyl peptidase-4

A (basically, COX converts AA to PG, and NSAIDS block COX)

A 39-year-old male presents to his primary care physician for his annual physical examination. Although he is in fine physical shape, his work as a construction superintendent is "very physical" and he reports back pain and occasional muscular pain. His physician recommends any of the variety of non-steroidal anti-inflammatory drugs available over the counter. These work by blocking the conversion of A. Arachidonic acid to prostaglandins by cyclooxygenase B. Arachidonic acid to prostaglandins by phospholipase A2 C. Arachidonic acid to thromboxane A2 by phospholipase A2 D. Phospholipids in the plasma membrane to arachidonic acid by cyclooxygenase E. Phospholipids in the plasma membrane to arachidonic acid by phospholipase A2

C (hydralazine is a common drug used for hypertension management, but a major side effect is drug-induced lupus, one of the main features of which is a maculopapular facial rash - other side effects include reflex tachycardia, immune mediated hemolysis, glomerulonephritis, and vasculitis)

A 40 year-old African American female visits your office for an annual checkup appointment. The patient has a history of hypertension refractory to thiazide diuretics and angiotensin converting enzyme (ACE) inhibitors. Blood pressure is 170/115 mmHg and BMI is 31 m/kg^2. You write a prescription for hydralazine. You instruct the patient to call your office is she experiences which of the following drug-related side effects? A. Cough B. Angioedema C. Maculopapular facial rash D. Seizure E. Weight loss

D (the patient was prescribed buspirone, which works at the 5HT1A receptor and is one of the first line medications for generalized anxiety disorder)

A 42-year-old female complains of feeling anxious and worrying about nearly every aspect of her daily life. She cannot identify a specific cause for these symptoms and admits that this tension is accompanied by tiredness and difficulty falling asleep. To treat this problem, the patient is prescribed the first-line therapy, a medication which notably lacks any anticonvulsant or muscle relaxant properties. This drug most likely acts at which of the following receptors? A. GABA receptor B. Alpha adrenergic receptor C. Glycine receptor D. 5HT-1A receptor E. Beta adrenergic receptor

E (long term corticosteroid use is associated with osteoporosis, along with many other side effects, including Cushingoid symptoms [obesity/weight gain, moon facies, hirsutism], muscular weakness, neutrophilia, hypoglycemia, and diastolic hypertension)

A 42-year-old female with a history of systemic lupus erythematous (SLE) has a 3-year history of daily prednisone (20 mg) use. Due to long-term prednisone use, she is at increased risk for which of the following? A. Hair loss B. Weight loss C. Pancreatic insufficiency D. Systolic hypertension E. Pathologic fractures

C (the patient has rheumatoid arthritis, a state of chronic inflammation caused by TNF-alpha - an inhibitor of TNF-alpha is a good idea, but the patient should first be screened for latent infection, such as TB [caused by Mycobacterium tuberculosis] to avoid serious problems)

A 43-year-old female presents with several years of pain and stiffness in her wrists and hands. Her symptoms have been getting progressively worse. She reports that her hands feel stiff for an hour or more after awaking in the morning. She undergoes subsequent testing, and is found to have anti-citrullinated protein antibodies. Her physician is concerned for rheumatoid arthritis, and discusses treatment with a biologic disease-modifying anti-rheumatic agent. Which of the following must be done before beginning treatment with a TNF inhibitor? A. Confirm diagnosis with a test for rheumatoid factor B. Confirm diagnosis with elevated ESR and CRP C. Test for latent mycobacterial infection D. Obtain a radiograph demonstrating osteophytes E. Obtain an exercise stress test

E (the answer is an MAO inhibitor, drugs used for depression that work by inhibiting monoamine oxidase, which metabolizes norepinephrine - severe hypertension follows the ingestion of large amounts of tyramine [found in wine, cheese, and other aged food products] due to the release of catecholamines and their extended action because the enzyme that metabolizes them is inhibited)

A 44-year-old female presents to the emergency department with severe hypertension. She does not have a history of high blood pressure. The previous night she attended a wine and cheese tasting. Which of the following drugs is most likely responsible for the hypertension? A. Bupropion B. Citalopram C. Haloperidol D. Nortriptyline E. Phenelzine

D (simvastatin is used to treat hyperlipidemia to lower LDL cholesterol, which is a risk factor for heart disease - it does this by inhibiting HMG-CoA reductase, which prevents the synthesis of cholesterol - side effects include myopathy, muscle weakness, and hepatotoxicity)

A 44-year-old male with a past medical history of ypertension, hyperlipidemia, and a 50-pack-year smoking history presents for a routine evaluation. Family history reveals that his father passed away from a myocardial infarction at 42 years old. He is subsequently started on a medication but returns 2 weeks later with complaints of lower leg pains and weakness. The medication that was most likely started is A. Aspirin B. Lisinopril C. Metoprolol D. Simvastatin E. Warfarin

B (a large Vd indicates that the drug is distributed widely throughout body tissues, meaning there is comparatively less of it in the plasma)

A 45-year-old female is on daily medication with notriptyline for depression. This drug is known to have a relatively large volume of distribution (Vd) and based on that it is most appropriate to state that A. A high percentage is bound to plasma proteins B. For a given dose, its plasma concentration is low C. It is ionized at physiological pH D. It is very potent E. Vd is close to her total plasma volume

C (the patient was given a tetracycline, likely doxycycline, for treatment of her cat scratch disease fever, or Bartonella henselae infection - the classic side-effect is photosensitivity, including phototoxic dermatitis - other side effects include nephro and hepatotoxicity, discoloration of teeth, and bone deformity of fetuses)

A 45-year-old female presents to her family medicine physician complaining of fever for 2.5 weeks, enlarged lymph nodes in her right axilla, and a small bump that formed when her recently adopted cat bit her 3 weeks ago. Based on her symptoms and history, the physician prescribed an antibiotic to treat her current illness. Several days later, she experienced erythema and itchiness on her shoulders, arms, and back after spending only several minutes a day in the sun. Which of the following is the mechanism of action of the antibiotic prescribed? A. Blocks the peptidyltransferase on the 50S ribosomal subunit B. Forms toxic free radicals that damage DNA C. Binds to the 30S ribosomal subunit preventing aminoacl-tRNA attachment D. Binds to PBP3 preventing the cross-linking of the peptidoglycan layer E. Blocks peptide translocation by binding to the 23S rRNA of the 50S ribosomal subunit

C (buspirone is an anxiolytic that is not a benzodiazepine, which reduces its abuse potential) (A; this is a short-acting benzodiazepine - good for panic attacks and acute anxiety, but not for generalized anxiety, and does have abuse potential) (B; this is an antidepressant that works well for smoking cessation) (D; this is a first generation antipsychotic, and not used for anxiety) (E; this is an MAOI, also not used for anxiety)

A 45-year-old female presents to her primary care physician and states that she is always worried about something and that these concerns are keeping her from concentrating on anything and damaging her interpersonal relationships. She does not want to take medication because she is afraid of becoming dependent. The drug, which will help her anxiety with the lowest risk of dependence, is A. Alprazolam B. Bupropion C. Buspirone D. Fluphenazine E. Phenelzine

D (the drug is metronidazole, which is approved for triple therapy of H pylori and causes a disulfiram like reaction when consumed with alcohol - this means you can expect levels of acetaldehyde in the blood to increase as well)

A 45-year-old female with a history of gastroesophageal reflux disease presents to her family physician with symptoms of epigastric pain right after a meal. The physician performs a urea breath test which is positive and the patient is started on appropriate medical therapy. Three days later at a restaurant, she experienced severe flushing, tachycardia, hypotension, and vomiting after her first glass of wine. Which of the following is the mechanism of action of the medication causing this side effect? A. Blocks the synthesis of the peptidoglycan layer B. Blocks protein synthesis by binding to the 50S ribosomal subunit inhibiting protein translocation C. Binds to the 30S ribosomal subunit preventing attachment of the aminoacyl-tRNA D. Forms toxic metabolites that damage bacterial DNA E. Inhibits the H+/K+ ATPase

A (acarbose inhibits the intestinal brush border enzyme alpha-glucosidase, which normally acts to break down starch and disaccharides into glucose - this delays sugar hydrolysis and absorption, leading to a decrease in postprandial glucose - this is also what is causing the patient's side effect of flatulence and diarrhea) (B; this is a GLP1 receptor agonist) (C; this is a second generation sulfonylurea) (D; this is an amylin analogue) (E; this is a thiazolidinedione)

A 45-year-old female with a past medical history of hypercholesterolemia and diabetes mellitus type 2 presents for new onset of diarrhea and increased flatulence. On exam you note paravertebral hypertonicity and tenderness from T6-T10. The patient states that her endocrinologist recently started her on a new medication for her uncontrolled hemoglobin A1v levels. Which of the following medications inhibits the enzyme alpha-glucosidase? A. Acarbose B. Exenatide C. Glipizide D. Pramlintide E. Rosiglitazone

B (the patient has meningitis with a positive India ink test, which is diagnostic for Cryptococcus neoformans, a serious fungal infection common in AIDS patients - the treatment of choice is a combination of amphoterecin B and flucytosine - the labs reveal an elevated BUN and creatinine, indicating renal toxicity, which is a side effect/toxicity of amphoterecin B administration - amphoterecin B's mechanism of action is to punch holes in the fungal cell membrane [increase permeability] by binding to ergosterol)

A 45-year-old patient with a history of HIV infection and a CD4+ count of 45/mm^3 presents with a headache, stiff neck, and impaired mental status. A cerebrospinal fluid sample is obtained by lumbar puncture, and an India ink stain performed on the sample reveals heavily encapsulated yeast. One week after initiation treatment, the patient has routine laboratory results drawn as shown. Sodium 138 mEq/L Potassium 2.8 mEq/L Chloride 100 mEq/L BUN 32 mg/dL Creatinine 1.5 mg/dL What is the mechanism of action for the medication that is most likely responsible for the abnormal laboratory results? A. disturbs the integrity of the fungal cell wall via inhibition of fungal (1-->3)-beta-D-glucan synthesis B. increases membrane permeability by binding to ergosterol in the fungal plasma membrane C. interferes with DNA synthesis making it a strong fungicidal agent D. inhibits ergosterol synthesis E. interferes with microtubule function

B (diphenhydramine, a 1st generation antihistamine, can cause severe sedation as a side effect due to significant CNS penetration, so it's a dangerous drug to give for allergies to a truck driver)

A 45-year-old truck driver complains of worsening seasonal allergies. His past medical history is significant for hypercholesterolemia, type II diabetes mellitus, and hypertension for which he takes a statin, metformin, and lisinopril. Which of the following medications should be avoided in this patient as therapy for his daytime allergies? A. Montelukast B. Diphenhydramine C. Loratadine D. Fexofenadine E. Fluticasone

E (metformin is absolutely contraindicated in patients with renal failure, due to a risk of renal tubular acidosis - an elevated serum creatinine suggests a decrease in GFR and possible renal failure)

A 45-year-old woman presents to your office with a serum glucose of 250 mg/dL and you diagnose diabetes mellitus type II. You intend to prescribe the patient metformin, but you decide to order laboratory tests before proceeding. Which of the following basic metabolic panel values would serve as a contraindication to the use of metformin? A. K+ > 4.0 B. Na+ > 140 C. HCO3- > 30 D. Glucose > 300 E. Creatinine > 2.0

A (bosentan is an endothelin receptor antagonist indicated for use in severe pulmonary artery hypertension)

A 45-year-old woman with history of systemic sclerosis presents with new onset dyspnea, which is worsened with moderate exertion. She also complains of chest pain. An ECG was obtained, and showed right-axis deviation. Chest x-ray showed right ventricle hypertrophy. Given the patient's history and presentation, right heart catheterization was performed, which confirmed the suspected diagnosis of pulmonary artery hypertension. It is decided to start the patient on bosentan. Which of the following describes the method of action of bosentan? A. Endothelin receptor antagonist B. Endothelin receptor agonist C. Phosphodiesterase type 5 inhibitor D. Calcium channel blocker E. Anticoagulant

A (metoclopramide is a D2 antagonist, the same as prochlorperazine) (B; this is a cannabinoid and a centrally acting antiemetic) (C; this is a centrally acting antiemetic that antagonizes substance P) (D; this is a centrally acting antiemetic that antagonizes 5-HT3 receptors) (E; cisapride acts through serotonin receptors to increase acetylcholine in the myenteric plexus - it is banned from the US for cardiotoxic effects)

A 46-year old diabetic woman presents with intense nausea for 2 weeks duration. She is subsequently diagnosed with diabetic gastroparesis and started on metoclopramide as an anti-emetic. Which of the following drugs has a similar mechanism of action as that of metoclopramide? A. Prochlorperazine B. Dronabinol C. Aprepitant D. Ondansetron E. Cisapride

C (the drug of choice for GERD is proton pump inhibitors, like omeprazole, which decrease the production of hydrochloric acid by inhibiting the H/K ATPases of parietal cells in the stomach)

A 46-year-old female presents to her primary care physician with dyspepsia, vague discomfort in the upper abdomen, increased flatulence, and a feeling of fullness and burning in her throat. She states that this happens on a regular basis, almost nightly, over the past few weeks. Physical exam reveals epigastric tenderness, a tender nodule in the left 5th intercostal space near the sternum, and paravertebral hypertonicity from T4-T9. The patient's BMI is 35. With appropriate laboratory tests run, she is prescribed a medication that is most efficacious in treating gastric-esophageal reflux disease (GERD). The mechanism of this medication is A. Antagonizing histamine receptors B. Blocking muscarinic receptors C. Inhibiting H/K ATPases D. Neutralizing stomach acids E. Stimulating prostaglandin receptors

E (tamoxifen is a selective estrogen receptor modulator that antagonizes estrogen in breast tissue [used for breast cancer] but enhances it in endometrial tissue - this creates a significant risk for endometrial hyperplasia and adenocarinoma)

A 46-year-old female presents with a primary complaint of irregular menstrual bleeding that has persisted for the last several months. Her medical history consists of 2 full-term pregnancies and invasive ductal carcinoma of the breast that was successfully treated one year ago. After an appropriate evaluation, she is given a diagnosis of endometrioid adenocarcinoma. Which of the following factors most likely increased this patient's risk for developing this cancer? A. Administration of raloxifene in the treatment of her previous breast cancer B. The patient is 46-years-old and has yet to enter menopause C. Combined oral contraceptive use from the age of 32 to 42 D. The patient's two previous pregnancies E. Tamoxifen regimen used to treat her previous breast cancer

D (the patient was given an ACE inhibitor, which causes renal efferent arteriolar vasodilation and a decrease in GFR in renally compromised patients - the symptomatic presentation of this patient reflects an underlying renal problem, such as bilateral renal artery stenosis)

A 46-year-old woman with a history of type II diabetes mellitus is started on lisinopril for newly diagnosed hypertension by her primary care physician. At a follow-up appointment several weeks later, she reports decreased urine output, and she is noted to have generalized edema. Her creatinine is elevated compared to baseline. Given her presentation, which of the following changes in renal arteriolar blood flow and glomerular filtration rate (GFR) have likely occurred? A. Renal afferent arteriole vasoconstriction; decreased GFR B. Renal afferent arteriole vasodilation; increased GFR C. Renal efferent arteriole vasoconstriction; increased GFR D. Renal efferent arteriole vasodilation; decreased GFR E. Renal efferent arteriole vasodilation; no change in GFR

B (A; fluoxetine is an SSRI and inhibits more serotonin reuptake) (C; this is the job of SSRIs and SNRIs, not MAOIs) (D; phenelzine is nonselective, and affects MAO-A and MAO-B equally) (E; phenelzine, as an MAOI, comes with dietary restrictions, including aged foods like wine, cheese, soy sauce, etc.)

A 49-year-old female visits her psychiatrist three months after an attempted suicide. Despite being placed on fluoxetine following the suicide attempt, she reports depressed mood, anhedonia, decreased sleep, decreased energy, and increased appetite. Her psychiatrist decides to taper the fluoxetine and do a trial of phenelzine. A practical reason to switch from fluoxtetine to phenelzine is that, compared to fluoxetine, phenelzine: A. inhibits serotonin reuptake at the synapse B. may be more efficacious for atypical, treatment-resistant depression C. will affect synaptic norepinephrine and serotonin reuptake D. will not affect MAO-B, leaving dopamine levels unchanged E. will not come with dietary restrictions

C (the kid is wearing scopolamine patches to treat his seasickness, but symptoms suggest that he overdosed - scopolamine is an anti-muscarinic agent, causing visual symptoms, dry skin, fever, and tachycardia - other antimuscarinic overdose symptoms are constipation and urinary retention, as well as dry mouth) (treatment is supportive with pheostigmine to inhibit AChE)

A 5-year-old boy is brought to the ED with fever and altered mental status. The boy was in his normal state of health until a few hours ago when he began to "not feel good" while on a deep-sea fishing trip as part of his family vacation. By the time that the family made it back to shore, the boy was experiencing blurred vision and confusion. In the ED his vitals were 101.6F, HR160, RR18, and O2 99% on room air. On exam, his skin is dry and flushed, and his abdomen is covered in small, round, stickers similar to that in the image. What gastrointestinal and urinary symptoms would you expect in this patient? A. Incontinence of stool; urine retention B. Incontinence of stool and urine C. Constipation; urine retention D. Constipation; incontinence of urine E. Incontinence of stool; no urinary symptoms

E (the patient has ingested ethylene glycol, which is converted to toxic metabolites by the enzymes alcohol dehydrogenase and aldehyde dehydrogenase - the ideal treatment is to block the first enzyme, which is what fomepizole does) (A; dimercaprol treats arsenic, lead, and mercury poisoning) (B; N-acetylcystein treats acetaminophen overdose) (C; ammonium chloride is used to acidify urine to treat overdose of basic compounds, like amphetamines) (D; flumazenil can be used to treat benzodiazepene or barbiturate overdose)

A 5-year-old boy is brought to the emergency room lapsing in and out of consciousness. The mother reports that 30 minutes ago, the young boy was found exiting the garage severely confused. A container of freshly spilled antifreeze was found on the garage floor. The next appropriate step would be to administer A. Dimercaprol B. N-acetylcysteine C. Ammonium chloride D. Flumazenil E. Fomepizole

E (both of these drugs are potassium sparing diuretics - amiloride and triamterene act by blocking ENaC, spironolactone acts by bocking the aldosterone-dependent Na/K exchange)

A 52-year-old Caucasian male presents to his primary care physician with complaint of shortness of breath and difficult breathing lying in bed at night, along with some dizziness, fatigue, and weakness. He also admits to a dry, hacking cough. Physical examination reveals pitting edema of his ankles, legs, and abdomen and a weight gain of 20 lbs over the past few months. The patient also admits to increased frequency of urination during the night. He is placed on a potassium sparing diuretic, and as such the physician needs to make a choice between A. Ethacrynic acid and acetazolamide B. Hydrochlorothiazide and amiloride C. Mannitol and furosemide D. Spironolactone and furosemide E. Spironolactone and triamterene

D (the patient has type II diabetes and needs the appropriate therapy - metformin is on the first line for type II diabetes treatment, and is not likely to cause weight gain, unlike the other drugs listed)

A 52-year-old woman comes to your clinic for her annual physical exam. She is obese, does not exercise, and regularly eats fried foods. A random blood glucose is 249 mg/dL. Her hemoglobin A1C is 9.5. Which of the following treatments would be weight neutral or cause weight loss in this patient? A. Glargine B. Glyburide C. Pyoglitazone D. Metformin E. Glipizide

A (sitagliptin is a dipeptidyl peptidase 4 [DPP4] inhibitor, which means it inhibits the degradation of endogenous incretins like GLP1 and GIP) (B; this is the mechanism of acarbose and miglitol) (C; this is the mechanism of thiazolidinediones) (D; this is the mechanism of sulfonylureas) (E; this is the mechanism of amylin mimetics)

A 53-year-old male presents to your office for a regularly scheduled check-up. The patient was diagnosed with type II diabetes mellitus two years ago. To date, diet, exercise, and metformin have failed to control his elevated blood glucose. Past medical history is also significant for hypertension. The patient does not smoke or use cigarettes. Laboratory values show a hemoglobin A1c (HbA1c) of 8.5%. You decide to add sitagliptin to the patient's medication regimen. Which of the following is the mechanism of action of sitagliptin? A. Inhibits degradation of endogenous incretins B. Inhibits alpha-glucosidases at the intestinal brush border C. Activates transcription of PPARs to increase peripheral sensitivity to insulin D. Depolarizes potassium channels in pancreatic beta cells E. Increases secretion of insulin in response to oral glucose loads and delays gastric emptying

B (the patient's side effect is niacin-induced flushing, probably because you prescribed him niacin to help improve his lipid profile - the flushing is caused by release prostaglandins and aspirin takes care of that no problem) (A, D; these are first-gen histamine antagonists used to treat allergic reactions, which this is not) (C; this is used to treat statin-induced myalgia) (E; acetaminophen is an analgesic, but won't fix flushing)

A 53-year-old man with a history of hypertension, hyperlipidemia, and obesity presents to you in clinic for a yearly physical. His current medication regimen includes a beta blocker, angiotensin converting enzyme inhibitor, and a statin. You review his recent lab work and note that despite being on a maximum statin dose, his LDL cholesterol remains elevated. You decide to prescribe another medication to improve his lipid profile. One month later, you receive a telephone call from your patient; he complains of turning bright red and feeling "scorching hot" every time he takes his medications. You decide to prescribe the which of the following medications to alleviate his symptoms: A. Diphenhydramine B. Aspirin C. Coenzyme Q10 D. Hydroxyzine E. Acetaminophen

E (nitrates, approved medication for stable angina, can cause headache, which is believed to be caused by vasodilation)

A 62-year-old Caucasian male receiving treatment for stable angina experiences intermittent throbbing headaches. What is the most likely cause? A. Transient ischemic attack B. Elevated creatine kinase C. Beta adrenergic inactivation D. Acute hemorrhage E. Vasodilation of cerebral arteries

E (the patient has elevated serum and urine uric acid concentrations - probenecid acts by decreasing the uptake of uric acid from the urine, therefore increasing the concentration - this can lead to kidney stones and is therefore not recommended for the patient's gout treatment)

A 54-year-old male has a history of gout complicated by several prior episodes of acute gouty arthritis and 3 prior instances of nephrolithiasis secondary to uric acid stones. He has a serum uric acid level of 11 mg/dL (normal range 3-8 mg/dL), a 24 hr urine collection of 1300 mg uric acid (normal range 250-750 mg), and a serum creatinine of 0.8 mg/dL with a normal estimated glomerular filtration rate (GFR). Which of the following drugs should be avoided in this patient? A. Naproxen B. Colchicine C. Allopurinol D. Indomethacin E. Probenecid

D (side effects of pioglitazone include weight gain, edema, hepatotoxicity, and heart failure - it should not be given to patients with heart or liver issues) (pioglitazone and its derivatives are drugs to reduce insulin resistance in DM2 patients, and are usually combined with another medication)

A 55-year-old male is hospitalized for acute heart failure. The patient has a 20-year history of alcoholism and was diagnosed with diabetes mellitus type 2 (DM2) 5 years ago. Physical examination reveals ascites and engorged paraumbilical veins as well as 3+ pitting edema around both ankles. Liver function tests show elevations in gamma glutamyl transferase and aspartate transaminase (AST). Of the following medication, which most likely contributed to this patient's presentation? A. Glargine B. Glipizide C. Metformin D. Pioglitazone E. Pramlintide

A (these are all side effects of cimetidine, the prototype drug of H2 blockers)

A 55-year-old man with peptic ulcer disease has been on drug therapy for more than 4 months. While on this regimen, he noticed changes in his bowel habits, increasing headaches, dizziness, skin rashes, loss of libido, and gynecomastia. The drug most likely responsible for these effects is. A. Cimetidine B. Famotidine C. Metronidazole D. Omeprazole E. Sucralfate

C (mannitol is an osmotic diuretic that increases plasma osmolarity - the intent of giving this drug is to increase the movement of fluid into the vascular space to lower intracranial pressure - however, this fluid shift may also cause pulmonary edema)

A 56-year-old male with history of CHF presents to a trauma center following a motor vehicle accident. On arrival, his Glasgow Coma Scale score is 8, and he is found to have increased intracranial pressure. Mannitol is administered. Which of the following side effects of the drug would you most likely observe in this patient? A. Blood dyscrasias B. Seizures C. Pulmonary edema D. Arrhythmias E. Restrictive cardiomyopathy

B (azithromycin, a macrolide, binds to the 50S ribosomal subunit, just like chloramphenicol) (A; this is a fluoroquinolone and targets DNA gyrase) (C; bacitracin inhibits cell wall synthesis by blocking the permeability of the cell membrane) (D; this aminoglycoside targets the 30S ribosomal subunit) (E; this attaches directly to the D-ala-D-ala cell wall structure)

A 56-year-old woman with no significant past medical history presents to her primary care physician with a cough. She notes that the cough began two weeks ago and has been worsening. Over the past several days, she has developed low-grade fevers, and the cough has become productive of green mucus. Her physician decides to treat her with a "Z-pack," which is a 5-day course of azithromycin, and orders a chest radiograph. Which of the following antibiotics has the same target site as azithromycin? A. Ciprofloxacin B. Chloramphenicol C. Bacitracin D. Gentamycin E. Vancomycin

E (sotalol, as a class III antiarrhythmic, lengthens the action potential, QT interval, and refractory period) (A, B; these are class IC antiarrhythmics which do not affect the length of action potential) (C; this is a class IB antiarrhythmic that shortens the duration of the action potential) (D; this ia class IV antiarrhythmic that decreases AV node conduction)

A 57-year-old Caucasian male presents to your office with heart palpitations and shortness of breath. On exam, he is tachycardic and his rhythm is irregularly irregular. He fails standard pharmacologic therapy and you refer to cardiology, where he is started on an antiarrhythmic medication. The action of this agent results in a longer action potential duration, an increased effective refractory period, and a longer QT interval. Which drug has been prescribed? A. Flecanide B. Propafenone C. Mexiletine D. Diltiazem E. Sotalol

B (sildenafil [viagra] is a selective inhibitor cGMP-specific phosphodiesterase type 5 - an inhibitor of conversion of cGMP leads to its accumulation within the cell)

A 57-year-old male presents with a primary complaint of erectile dysfunction. After proper evaluation, the patient is started on daily administration of sildenafil. This medication directly causes accumulation of which of the following intracellular mediators? A. Ca2+ 2. cGMP 3. AMP 4. NO 5. ANP

B (finasteris is a 5-alpha-reductase inhibitor, which inhibits the peripheral conversion of testosterone to dihydrotestosterone [DHT] - BPH and alopecia are associated with elevated levels of 5-alpha-reductase and DHT and can be treated with finasteride)

A 57-year-old presents to your clinic complaining of baldness. He is overweight, has been diagnosed with BPH, and is currently taking atorvastatin for hyperlipidemia. The patient has tried several over-the-counter products for hair-loss; however, none have been effective. After discussing several options, the patient is prescribed a medication to treat his baldness that has the additional benefit of treating symptoms of BPH as well. Synthesis of which of the following compounds would be expected to decrease in response to this therapy? A. GnRH B. DHT C. LH D. Testosterone E. FSH

D (esmolol is a class II antiarrhythmic and a beta blocker - glucagon is an antidote for beta blocker toxicity)

A 57-year-old woman with a history of diabetes and hypertension accidentally overdoses on antiarrhythmic medication. Upon arrival in the ER, she is administered a drug to counteract the effects of the overdose. Which of the following matches an antiarrhythmic with its correct treatment in overdose? A. Quinidine and insulin B. Encainide and epinephrine C. Propafenone and glucose D. Esmolol and glucagon E. Sotalol and norepinephrine

C (ACE inhibitors are first-line drugs for hypertension for patients with history of MI because ACE inhibitors are also cardioprotective and reduce cardiac remodeling - a beta blocker would also likely be included)

A 67-year-old man with a history of diabetes mellitus type II and a previous myocardial infarction presents to your office for a routine examination. His blood pressure is found to be 180/100 mmHg. Which drug is the first-line choice of treatment for this patient's hypertension? A. Amlodipine B. Hydrochlorothiazide C. Lisinopril D. Prazosin E. Isoproterenol

E (theophylline increases catecholamine levels and blocks the action of adenosine to reverse bronchospasm and improve COPD - theophylline is metabolized by the P450 system - the patient likely had a fungal infection between his toes, for which he was probably given ketoconazole, or another azole, which inhibit the P450 system - this causes the theophylline to accumulate and induce toxicity, which includes nausea, vomiting, diarrhea, and abdominal pain - if the diarrhea persists, the toxicity can lead to electrolyte disturbance)

A 58-year-old African-American male presents to the community health clinic with the complaint of nausea, vomiting, and diarrhea for the past four days. He has a history of chronic obstructive pulmonary disease (COPD) and physical examination reveals a barrel-shaped chest and a prolonged exhalation phase with decreased bibasilar breath sounds. He states he has been taking a medication for his COPD every day for the past two years. However, his primary care physician started him on a pill last week due to an infection he had between his toes. What is the most likely medication this person has been taking for his COPD? A. Albuterol B. Montelukast C. Prednisone D. Salmeterol E. Theophylline

B (A; Warfarin inhibits vitamin-K dependent synthesis of factors II, VII, IX, and X, as well as proteins C and S) (C; aspirin irreversibly inhibits COX-1 and COX-2) (D; clopidogrel and prasugrel antagonize ADP receptors on platelets, and prasugrel has an increased risk of bleeding compared to clopidogrel)

A 58-year-old Caucasian male with a history of peripheral vascular disease is admitted to the hospital with a painful, pulseless foot. He is prescribed antiplatelet and andicoagulant drugs. Which of the following correctly pairs a drug with its characteristic? A. Warfarin: directly inhibits thrombin B. Heparin: activates antithrombin 3 C. Aspirin: reversibly inhibits COX-1 D. Clopidogrel_ antagonizes ADP receptors on endothelial cells E: Prasugrel: reduced risk of bleeding compared to other drugs

E (this is the only drug listed that can treat depression with a low risk of sexual side effects)

A 58-year-old caucasian man comes into the psychiatry office complaining of depressed mood. For the past two months, he has had trouble sleeping, even though he feels tired throughout the day, and decreased appetite. During this time, he's had little interest in his former hobbies. His only other medical complaint is pre-existing sexual dysfunction. Which of the following is the most appropriate treatment for this patient's depression? A. Duloxetine B. Sertraline C. Amoxapine D. Paroxetine E. Bupropion

C (epinephrine activates a1, a2, b1, and b2 receptors, but in the presence of a beta blocker like propranolol, only the alpha effects will manifest, which means vasoconstriction and increased blood pressure, resulting in a reflex bradycardia)

A 58-year-old male is diagnosed with hypertension and started on daily propranolol after failing antihypertensive therapy with other medications. Three months later, his blood pressure appears to be adequately controlled on this therapy. While working in his garden one afternoon, the patient is stung by a wasp and experiences a severe anaphylactic reaction that is subsequently treated with epinephrine. Which of the following effects would be expected upon administration of this epinephrine treatment? A. Decreased bronchodilation B. Increased motility of the gastrointestinal tract C. Decreased heart rate D. Decreased blood pressure E. Increased heart rate

E (chlorpromazine is a first generation antipsychotic that has a unique side effect of corneal deposits, as well as light sensitivity and a potential for blindness - other side effects match those of the other first generation antipsychotics, such as QT elongation, hypotension, akathisia, and tardive dyskinesia)

A 58-year-old male presents to his primary care doctor with the complaint of vision changes over the last several months. The patient's past medical history is notable for schizophrenia which has been well-controlled for the last 25 years on chlorpromazine. Which of the following is likely to be seen on ophthalmoscopy? A. Retinitis pigmentosa B. Macular degeneration C. Glaucoma D. Retinal hemorrhage E. Corneal deposits

E (the diagnoses are CHF and HTN, the side effects are cough and hyperkalemia - the only drug of the listed options that causes those side effects and is perscribed for those conditions is lisinopril, an ACE inhibitor)

A 58-year-old male with a history of congestive heart failure and hypertension comes to you with the chief complaint of new-onset cough as well as increased serum potassium in the setting of a new medication. Which of the following medications is most likely responsible for these findings? A. Furosemide B. Metoprolol C. Amiodarone D. Digoxin E. Lisinopril

C (the patient has congestive heart failure - spironolactone is a mineralocorticoid antagonist that can cause gynecomastia as an adverse effect - other good drugs to perscribe this patient include beta blockers, ACE inhibitors, and ARBs, although none of these are known to cause gynecomastia) (A; this would be the effect of beta blockers) (B; this would be the effect of ACE inhibitors) (D; this would be the effect of loop diuretics) (E; this would be the effect of mannitol, which is contraindicated in CHF)

A 58-year-old man who had been complaining of increased shortness of breath with exertion and paroxysmal nocturnal dyspnea was started on a medical regimen to help stabilize his condition and relieve his fluid retention. Though his symptoms were improving, he noticed bilateral breast enlargement and returned to the clinic. The drug responsible for his breast enlargement also functions to do which of the following? A. Increase the PR interval on EKG B. Inhibit angiotensin converting enzyme C. Bind mineralcorticoid receptors D. Inhibit the Na+/K+/2Cl- triple transporter in the thick ascending limb of the loop of Henle E. Increase tubular fluid osmolarity

E (this patient likely has a stomach ulcer from NSAID abuse - aspirin is an irreversible Cox1 and Cox2 inhibitor - Celecoxib would have been a selective Cox2 inhibitor [the other choices are nonselective] and would have prevented the ulcer)

A 59-year-old male with a 1-year history of bilateral knee arthritis presents with epigastric pain that intensifies with meals. He has been self-medicating with aspirin, taking up to 2,000 mg per day for the past six months. Which of the following medications, if taken instead of aspirin, could have minimized his risk of experiencing this epigastric pain? A. Ketorolac B. Indomethacin C. Naproxen D. Ibuprofen E. Celecoxib

D (A is a subsance P antagonist, B is a cannbinoid, C and E are D2 antagonists)

A 67-year-old woman is being treated for metastatic ovarian cancer with cisplatin and cyclophosphamide. To prevent nausea and vomiting, she is given an agent that selectively antagonizes 5-hydroxytryptamine-3 receptors. Which of the following drugs is this patient most likely taking? A. Aprepitant B. Dronabinol C. Metoclopramide D. Ondansetron E. Prochlorperazone

C (this question is basically just asking if you know how glucocorticoids like prednisone exert their immunosuppressive effect - the answer is that they prevent the leukocytes from extravasating into the tissues, thus remaining in the circulating blood - this is called demargination)

A 59-year-old male with a history of chronic obstructive pulmonary disease (COPD) presents for follow-up from a recent emergency department visit. He states that he was diagnosed with acute exacerbation of COPD and provided prednisone, ipratropium, and an albuterol inhaler. A repeat blood count is performed revealing a severe neutrophilic leukocytosis. This is most likely due to A. collapse of alveoli B. constitutively active tyrosine kinase C. demargination of neutrophils D. exotoxin release E. reactive proliferation of myeloid cells

E (cough is a very likely side effect of ACE inhibitors, which is an indication to change the ACE inhibitor to an angiotensin receptor blocker instead) (A; cough is not a likely side effect of an SSRI) (B; giving this patient a beta blocker is not a good idea given the history of asthma) (C; amlodipine is not preferred, since ACE inhibitors and ARBs are preferred for diabetic hypertensive patients) (D; cough is not a likely side effect of statins)

A 59-year-old man presents to general medical clinic for his yearly checkup. He has no complaints except for a dry cough. He has a past medical history of type II diabetes, hypertension, hyperlipidemia, asthma, and depression. His home medications are sitagliptin/metformin, lisinopril, atorvastatin, albuterol inhaler, and citalopram. His vitals signs are stable, with blood pressure 126/79 mmHg. Hemoglobin A1C is 6.3%, and creatinine is 1.3 g/dL. The remainder of his physical exam is unremarkable. If this patient's cough is due to one of the medications he is taking, what would be the next step in management? A. Change citalopram to escitalopram B. Change lisinopril to metoprolol C. Change lisinopril to amlodipine D. Change atorvastatin to to lovastatin E. Change lisinopril to losartan

C (antidiarrheal agents are contraindicated in diarrheal illness of children due to the risk of developing toxic megacolon)

A 6-year old African-American boy is brought to the pediatrician by his mother because of a 2-day history of moderate, watery, non-bloody diarrhea. He has a low grade fever. Laboratory studies show hematocrit 44%, hemoglobin 18 g/dL, WBC count 13k/mm^3, platelet count 300k/mm^3. Which of the following is absolutely contraindicated in the treatment of this patient's illness? A. Acetaminophen B. Ciprofloxacin C. Loperamide D. Penicillin E. Fluid restoration

C (glipizide is a second generation sulfonylurea that triggers the release of insulin from pancreatic beta cells - C-peptide levels directly correlate with the release of endogenous insulin) (glipizide is only given to treat type II DM, and is contraindicated in sulfa drug allergies)

A 60-year-old African-American female presents to your office complaining of dysuria, paresthesias, and blurry vision. Her body mass index is 37.2 kg/m2. Which of the following drugs would most significantly increase the levels of C-peptide in the blood when administered to this patient? A. Metformin B. Insulin C. Glipizide D. Acarbose E. NPH

C (atenolol is selective for beta1 receptors, so it cannot act on the peripheral vasculature, which has beta2 receptors - atenolol treats hypertension by reducing the workload of the heart, not by affecting the vasculature) (B; the release of renin is mediated by beta receptors) (E; this is a direct effect of atenolol, if it is administered to the eye)

A 60-year-old Caucasian female with hypertension is treated with atenolol. Which of the following is NOT a direct effect of atenolol? A. Decreased cardiac contractile force B. Decreased renin levels C. Increased peripheral vasodilation D. Decreased heart rate E. Decreased intraocular pressure

A (this is herpes simplex encephalitis - the symptoms reflect an increase in intracranial pressure, which progresses to change in mental status, fever, and seizure activity - on physical exam, the healing sores around the lips indicate a recent flare of herpes simplex activity, causing the encephalitis - the only appropriate antiviral agent on the list is acyclovir) (B; this is for systemic fungal disease) (C; this is for broad spectrum meningitis treatment) (D; this is for cancer) (E; this is for toxoplasmosis)

A 60-year-old male presents with a new onset of headache and the feeling of great pressure in his head. Past medical history reveals an upper respiratory infection two weeks ago. Physical examination reveals healing sores on his lips. Magnetic resonance imaging is ordered as shown. After admission and a lumbar puncture, the patient develops confusion, a fever, and inters an ictal state. The most appropriate medication is A. Acyclovir B. Amphotericin C. Ceftriaxone D. Chemotherapeutic agents E. Pyrimethamine and sulfadiazine

B (the diagnosis is acute bacterial prostatitis - in an older patient, the most common cause is E coli - appropriate treatments would include doxycycline, ciprofloxacin, or TMP/SMX, which are active against E coli)

A 60-year-old male presents with fever, chills, and a burning sensation with urination. His symptoms started yesterday. He also complains of lower back pain. Rectal examination reveals an enlarged, tender prostate. Urinalysis is positive for bacteria. Which of the following is the best initial drug therapy? A. Ceftriaxone B. Ciprofloxacin C. Doxazosin D. Finasteride E. Penicillin

A (the patient has symptoms of acute psychosis, a possible side effect of the increased dopamine in her brain, from her primary Parkinson's treatment of levo/carbidopa)

A 61-year-old female with no prior medical history is diagnosed with Parkinson disease. She started on a pharmacologic treatment regimen, but shortly after beginning this therapy she is brought to the emergency room by her concerned husband who describes a dramatic change in behavior. He reports that she was speaking to people in an empty room and was becoming increasingly agitated. Which of the following medications is the most likely cause of these symptoms? A. Levodopa/carbidopa B. Amantadine C. Selegiline D. Benztropine E. Entacapone

D (acetazolamide is a carbonic anhydrase inhibitor, meaning it blocks the reabsorption of bicarbonate in the proximal tubule of the nephron - this raises urine pH and makes it alkaline) (A; this is true for loop diuretics) (B; this is true for thiazide diuretics) (C; aldosterone acts on the H/K ATPase in the alpha cells, but this acidifies the urine, rather than alkalinizes it) (E; bicarbone is secreted, not reabsorbed, by the beta cells)

A 61-year-old male is given acetazolamide to treat open-angle glaucoma. Upon diuresis, his urine is found to be highly alkaline. Which of the following accounts for the alkaline nature of this patient's urine? A. Inhibition of chlorine reabsorption in the thick ascending loop of Henle B. Inhibition of chlorine reabsorption in the distal convoluted tubule C. Inhibition of acid secretion in alpha-intercalated cells D. Inhibition of bicarbonate reabsorption in the proximal tubule E. Inhibition of bicarbonate reabsorption in beta-intercalated cells

C (the patient has uncomplicated constipation - the first line of treatment should be increasing fluid intake, and perhaps a bulk-forming laxative, something with a lot of fiber, like psyllium)

A 62-year-old female presents to her primary care physician with a chief complaint of constipation that has become more bothersome over the four months. She states that she had prior issues with constipation complicated by hemorrhoids 10 years ago. She denies any pain with defecation or blood in her stool. The patient reports having just had a colonoscopy 3 months ago with normal results. Her vital signs at this visit are within normal limits. Physical examination, including a rectal exam, does not reveal any abnormalities. The physician instructs the patient to increase her fluid and dietary fiber intake. In addition to these recommendations, which of the following agents would be most appropriate for the patient's constipation at this time? A. Bisacodyl B. Polyethylene glycol C. Psyllium D. Docusate E. Senna

A (metoclopramide is a dopamine antagonist, used for gastroparesis and as an antiemetic, but may cause extrapyramidal and Parkinsonian symptoms - a dopamine antagonist may be able to increase secretion of prolactin [which is inhibited by dopamine], as well as contraindicated in Parkinsonism)

A 63-year-old man presents to general medical clinic with involuntary, repetitive body movements. He reports that without trying to move his mouth, his lips begin smacking and puckering. He is quite concerned. He has a long history of diabetes mellitus complicated by diabetic gastroparesis, for which he is being medicated. You suspect his symptoms may be a side effect of this medical therapy. What is another indication and contraindication for the medication prescribed to the patient? A. Indication: decreased milk letdown during nursing, Contraindication: Parkinson's disease B. Indication: diarrhea; Contraindication: small bowel obstruction C. Indication: hypertension; Contraindication: Alzheimer's disease D. Indication: hyperglycemia;Contraindication: nausea E. Indication: diabetic foot ulcers; Contraindication: anxiety

D (amiodarone is a cool drug because it has traits of all the classes of antiarrhythmics [officially classified as class III though] and can prolong the QT interval without increasing the risk of torsades - it also has the cool side effects of blue-gray skin discoloration, pulmonary fibrosis, corneal deposits, hepatotoxicity, photosensitivity, and thyroid dysfunction) (A; sotalol is class III, increases both PR and QT, and predisposes to torsades) (B; under therapeutic dose, digoxin doesn't affect the QT, and under toxic dose, it shortents it) (C; esmolol is class II, very short acting and does not affect the QT) (E; quinidine is class IA, all of which predispose to torsades)

A 63-year-old man with a history of hypertension and atrial fibrillation is brought into the emergency room and found to have a ventricular tachyarrhythmia. Ibutilide is discontinued and the patient is switched to another drug that also prolongs the QT interval but is associated with a decreased risk of torsades de pointes. Which drug was most likely administered in this patient? A. Sotalol B. Digoxin C. Esmolol D. Amiodarone E. Quinidine

E (cilostazol inhibits phosphodiesterase III, causing increased cAMP within platelets leading to arterial vasodilation and decreased platelet aggregation - it's a first line medication for the treatment of PAD)

A 63-year-old mane is aiming to improve his health by eating a well-balanced diet, walking daily, and quitting smoking following a 45-year smoking history. While on his daily walks, he notices a strong cramping pain in his calves that consistently appears after a mile of walking. He sees his physician and a diagnosis of peripheral artery disease with intermittent claudication is made. To improve his symptoms, cilostazol is prescribed. What is the mechanism of action of this medication? A. Irreversible cyclooxygenase inhibitor B. Glycoprotein IIb/IIIa inhibitor C. Thromboxane synthase inhibitor D. Adenosine diphosphate receptor inhibitor E. Phosphodiesterase inhibitor

A (adenosine increases the potassium shift out of atrial myocardial and AV nodal cells, shortening phase 3 on pacemaker curves and hyperpolarizing pacemaker cells - at higher doses, adenosine decreases conduction velocity, prolongs refractory period, and decreases automaticity in the AV node - the risk of toxicity is low due to its short duration of action, but it may cause flushing, chest pain, and hypotension)

A 64-year-old Asian man presents to the emergency department with complaints of chest pain and palpitations. An electrocardiogram shows rapid rate, regular rhythm and narrow QRS complexes with no discernible P waves. These findings are suggestive for supraventricular tachycardia. Vagal manuevers are attempted but are unsuccessful. Of the following, the medication that would create a slower passage at the AV node would be A. Adenosine B. Dofetilide C. Epinephrine D. Lidocaine E. Nifedipine

D (this patient is on sildenafil [viagra] for erectile dysfunction - the little blue pill has the classic side effect of a little blue vision) (the mechanism of action of sildenafil is the inhibition of phosphodiesterase 5 [PDE5] which normally decreases cGMP)

A 65-year-old Caucasian male presents to his primary care physician to establish continued care. His medical history includes diabetes mellitus, hypertension, peripheral vascular disease, erectile dysfunction, and cataracts. He is currently taking metformin, glipizide, metoprolol, hydrochlorothiazide, and sildenafil. His physician asks him if he is currently experiencing any problems with his medications. His only complaint is that he has noticed a slight blue tint to his visual field. Fundoscopic exam is unremarkable for eye pathology. His change in vision is most likely due to the medication he takes for his A. Diabetes mellitus B. Hypertension C. Peripheral vascular disease D. Erectile dysfunction E. Cataracts

D (anastrozole is an aromatase inhibitor, which prevent peripheral conversion of androgens to estrogens, to treat breast cancer - inhibiting the synthesis of estrogen removes the protective effects that estrogen has in women, including bone mineral density, leading to osteoporosis - other symptoms would be hot flashes, vaginal dryness/atrophy, arthralgias, and increased risk for coronary events)

A 65-year-old female is diagnosed with invasive ductal carcinoma of the breast. After a lumpectomy with a negative axillary lymph node dissection and subsequent radiation therapy, the patient is started on adjuvant chemotherapy with anastrozole. Which of the following is an expected side effect of this medication? A. Increased risk for endometrial carcinoma B. Decreased LDL and increased HDL levels C. Mental confusion progressing to more severe dementia D. Increased risk for osteoporosis E. Excessive weight loss

C (furosemide is a loop diuretic that inhibits the Na/K/Cl symporter) (A is a carbonic anhydrase inhibitor) (B is an endogenous hormone secreted by the adrenal gland) (D inhibits the Na/Cl cotransporter in the distal convoluted tubule) (E is an aldosterone antagonist and a potassium sparing diuretic that inhibits the reabsorption of sodium ions in the collecting duct)

A 65-year-old male presents to his primary care physician for an annual check-up. He states that he has been eating well and that he has lost ten pounds since his last visit. You are concerned because his blood pressure has remained high. His hemoglobin A1C is 8.2%, his blood pressure is 141/93 mmHg, and his heart rate is 90/min. He was recently prescribed a blood pressure medication that causes the inhibition of the sodium-potassium-chloride symporter of the kidney. This medication is most likely A. Acetazolamide B. Aldosterone C. Furosemide D. Hydrochlorothiazide E. Spironolactone

E (of the listed drugs, only dobutamine is a selective beta 1 antagonist) (A; nonselective alpha and beta agonist) (B; phosphodiesterase inhibitor) (C; nonselective beta agonist) (D; nonselective alpha1, 2, and beta1 agonist, though preferentially acts on alphas)

A 65-year-old male with a history of CHF presents to the emergency room with shortness of breath, lower leg edema, and fatigue. He is diagnosed with acute decompensated congestive heart failure, was admitted to the CCU, and treated with a medication that targets beta-1 adrenergic receptors preferentially over beta-2 adrenergic receptors. The prescribing physician explained that this medication would only be used temporarily as its efficacy decreases within one week due to receptor downregulation. Which of the following was prescribed? A. Epinephrine B. Milrinone C. Isoproterenol D. Norepinephrine E. Dobutamine

E (the 'osin' drugs are alpha blockers, meaning they relax smooth muscle through inhibition of alpha 1 receptors - this includes reduction of vascular resistance and blood pressure by extension, and also smooth muscle relaxation in the bladder and prostate to encourage urination in patients with benign prostatic hyperplasia)

A 65-year-old male, who has been on furosemide to treat his hypertension, is diagnosed with benign prostatic hyperplasia. His physician decides to have the patient stop taking furosemide and begin taking a new drug that should treat both his hypertension and the problems caused by his enlarged prostate. The new drug the physician prescribed is most likely A. Clonidine B. Enalapril C. Finasteride D. Losartan E. Terazosin

B (of the given options, only Ketamine interacts at the NMDA receptor)

A 65-year-old man with metastatic lung cancer has been experiencing severe, unremitting pain. He has required escalating doses of oral morphine, but is now having dose limiting side-effects. His pain management team recommends using a medication that can reduce his opioid need through interaction with the NMDA-receptor. Which of the following was the most likely recommended agent? A. Propofol B. Ketamine C. Fentanyl D. Ketorolac E. Midazolam

B (the patient may be taking a phosphodiesterase inhibitor for his erectile dysfunction, which if combined with a nitrate, could be lethal - seriously, do not give anyone nitrates if you suspect they're on Viagra - they will literally die)

A 66-year-old gentleman presents to a new primary care physician to establish care after a recent relocation. His past medical history is significant for gout, erectile dysfunction, osteoarthritis of bilateral knees, mitral stenosis, and diabetic peripheral neuropathy. He denies any past surgeries along with the use of any tobacco, alcohol, or illicit drugs. He has no known drug allergies and cannot remember the names of the medications he is taking for his medical problems. He states that he has recently been experiencing chest pain with strenuous activities. What part of the patient's medical history must be further probed before starting him on a nitrate for chest pain? A. Gout B. Erectile dysfunction C. Arthritis D. Mitral stenosis E. Diabetic peripheral neuropathy

A (the notable finding on the EKG is peaked T waves, caused by hyperkalemia and is responsible for the patient's palpitations [arrhythmia] - given his diagnoses of hypertension and chronic kidney disease, he was likely prescribed lisinopril, an ACE inhibitor that can treat hypertension without acting at the kidneys, and causes hyperkalemia)

A 66-year-old male with a history of diabetes, hypertension, and chronic kidney disease presents to the emergency department with severe fatigue, muscle weakness, and palpitations. He states that it seemed to start yesterday and has progressed since that time. The emergency physician obtains an EKG as shown. The patient comments that a few days ago he visited his primary care physician during which some of his blood pressure medications were changed. Which of the following medications was most likely added during that visit? A. Lisinopril B. Furosemide C. Hydrochlorothiazide D. Nifedipine E. Hydralazine

A (gentamicin, an aminoglycoside, is notable ototoxic - furosemide, a loop diuretic, is also - combining the two therapies increases the risk of ototoxic side effects such as tinnitus and hearing loss) (B; loop diuretics are actually potassium wasting, and wouldn't lead to hyperkalemia) (D; this is "red man syndrome", an infusion related side effect of vancomycin) (E; this is classically a side effect of antibiotics like clindamycin, causing an overgrowth of Clostridium difficile)

A 67-year-old female is admitted to the hospital with enterococcus endocarditis and is treated with penicillin and gentamicin. During her admission, she develops worsening pulmonary edema secondary to valvular insufficiency and requires therapy with IV furosemide. This patient is most likely to experience which of the following adverse reactions to her current pharmacological treatment? A. Ringing in the ears and impaired hearing B. Hyperkalemia leading to possible cardiac arrhythmia C. Anemia and chronic fatigue D. Diffuse flushing and redness over body surface E. Pseudomembranous colitis and diarrhea

B (first line medication for hypertension nowadays is ACE inhibitors - a potential side effect is first-dose hypotension)

A 67-year-old gentleman with a history of poorly controlled diabetes presents to his primary care physician for a routine examination. He is found to be hypertensive on physical exam and is started on a medication that is considered first-line therapy for his condition. What should the physician warn the patient about before the patient takes his first dose of the medication? A. Hypertensive episodes B. Hypotensive episodes C. Hyperthermic episodes D. Hypothermic episodes E. Anuric episodes

E (the patient has temporal/giant cell arteritis which has progressed to ophthalmic artery involvement - the treatment is glucocorticoids, which have a side effect of neutrophilia by demargination [meaning that the neutrophils are high in the blood because they cannot extravasate into the tissues - this is because of an absence of adhesion factors in the endothelium, mediated by the steroids])

A 67-year-old woman presents to the emergency room with headache and vision loss. The patient reports several weeks of jaw pain, neck, and temple pain with intermittent fevers. This morning she woke up and she couldn't see out of her left eye. She is started on medication and spends the night in the emergency room. On admission, her white count was 8k. A follow up complete blood count reveals a white count of 18.2k with a neutrophilic predominance. What medication was most likely given to the patient? A. Lisinopril B. Atorvastatin C. Metformin D. Penicillin E. High dose steroids

C (the patient was likely given fursemide, a loop diuretic which acts on the Na/K/2Cl cotransport system in the thick ascending limb - furosemide can cause hypocalcemia and hypomagnesemia) (A; this is the site of action of acetazolamide and mannitol) (B; this is not a common site of action for diuretics) (D; this is where thiazide diuretics act) (E; this is where potassium-sparing diuretics act)

A 68-year-old male with congestive heart failure recently had his medication regiment adjusted to better control his hypertension. Three weeks later, laboratory analysis shows his serum calcium and magnesium levels have both decreased. The diuretic used in this patient acts predominantly on which nephron segment? A. Proximal tubule B. Descending loop of Henle C. Thick ascending loop of Henle D. Distal tubule E. Cortical collecting duct

D (acetylcholine causes smooth muscle contraction of the airway, as in asthma, so an antimuscarinic drug would be indicated for the patient)

A 7-year-old boy is brought to your office with complaints of wheezing and dyspnea. Laboratory work reveals eosinophilia and positive skin tests for allergens. Which of the following types of drugs would be an effective treatment in this patient? A. Beta-2 antagonist B. Histone deacetylase inhibitor C. VEGF inhibitor D. Muscarinic antagonist E. Anticoagulant

A (from the elevated INR and past history of MI and AFib, assume the patient is on warfarin - a positive head CT for bleed would be indicative of a need to transfuse blood and platelets, but since there is no evidence of a bleed, the correct therapy is vitamin K, to counteract the supratherapeutic effect of the patient's warfarin) (B; cryoprecipitate is used to treat fibrinogen and factor VIII deficiencies) (C; protamine is used to reverse the effects of heparin)

A 70-year old Caucasian male presents to the emergency room for a fall. His past medical history is significant for myocardial infarction and atrial fibrillation. His home medications are unknown, and the patient's head CT is shown. Laboratory results reveal an INR of 6. Which of the following is the most appropriate pharmacologic therapy for this patient? A. Vitamin K B. Cryoprecipitate C. Protamine D. Platelet transfusion E. Fresh frozen plasma

A (the patient is suffering from post-menopausal osteoporosis - the best treatment is a bisphosphonate that inhibits bone resorption)

A 70-year-old female presents to her geriatrician with a complaint of back pain which started after picking up a large back of groceries. The woman is thin and apparently in good health otherwise. Plain radiography of her lumbar region is obtained and reveals a compression fracture. Laboratory results are unremarkable. Bone densitometry is assessed by DEXA scan and reveals osteoporosis. The drug which will most likely slow the progression of her condition is A. Alendronate B. Corticosteroids C. Phosphate D. Plicamycin E. Progesterone

B (verapamil is a calcium-channel blocker to decrease the flow of calcium into myocardial cells and decrease their conduction - therefore verpamail has a negative inotropic effect and is contraindicated in patients with CHF or low ejection fractions)

A 70-year-old female with a past medical history of myocardial infarction and congestive heart failure (CHF)with a low ejection fraction has recently been prescribed verapamil. The most appropriate statement regarding this drug is that it A. decreases rehospitalizations in patients with CHF B. increases mortality in patients with CHF C. increases heart rates in patients with CHF D. increases anginal symptoms in patients with coronary artery disease E. increases the formation of methemoglobin

C (the patient is on prazosin, an alpha blocker, and isosorbide mononitrate, a nitrate - both are blood pressure reducing medications, and can cause orthostatic hypotension - enalapril and carvedilol may exacerbate this as well)

A 71-year-old man with a history of BPH is hospitalized for an ST-elevation myocardial infarction and undergoes percutaneous coronary intervention. Upon discharge, he is prescribed aspirin, clopidogrel, prazosin, isosorbide mononitrate, carvedilol, enalapril, and atorvastatin. He is scheduled to follow up with his primary care provider 1 week after discharge. Which of the following is the most likely complication of his drug regimen? A. Torsades de pointes B. Drug-induced hepatitis C. Orthostatic hypotension D. Agranulocytosis E. Seizure

B (captopril is an ACE inhibitor, triamterene is a potassium-sparing diuretic - in combination, there is high risk of hyperkalemia)

A 72-year-old anthropologist with long-standing hypertension visits your office for a routine exam. You notice an abnormality on his laboratory results caused by his regimen of captopril and triamterene. What abnormality did you most likely find? A. Hypercalcemia B. Hyperkalemia C. Hypernatremia D. Thrombocytopenia E. Anemia

E (warfarin is metabolized by the cytochrome P450 system, which is inhibited by drugs like valproic acid, therefore decreasing the metabolism of warfarin, and extending its therapeutic effects such as increasing INR) (A, B, C, and D are CP450 activators, and would decrease warfarin's effect, increasing INR)

A 72-year-old woman with a history of atrial fibrillation on warfarin, diabetes, seizure disorder and recent MRSA infection is admitted to the hospital. She subsequently begins therapy with another drug and is found to have an elevated INR. Which of the following drugs is likely contributing to the patient's elevated INR? A. Phenobarbital B. Glipizide C. Rifampin D. Carbamazepine E. Valproic acid

C (carvedilol is a beta blocker - since beta receptors usually are Gs to raise cAMP, carvedilol would therefore decrease cAMP) (nitroglycerin is a nitrate, which increases cGMP)

A 73-year-old man presents to his primary care physician with chest pain. He noticed the pain after walking several blocks, and the pain is relieved by sitting. On exam, he has a BP 155/89 mmHg, HR 79 bpm, and T 98.9 F. The physician refers the patient to a cardiologist and offers prescriptions for carvedilol and nitroglycerin. Which of the following describes the mechanism or effects of each of these medications, respectively? A. Increased cAMP; Increased cAMP B. Increased contractility; Decreased endothelial nitrous oxide C. Decreased cAMP; Increased cGMP D. Decreased cGMP; Increased venous resistance E. Increased heart rate; Decreased arterial resistance

C (while norepenephrine does act on alpha1, alpha2, and beta1 receptors, remember that it prefers the alpha receptors) (norepinephrine is a good push to normalize blood pressure in hypotensive patients, such as this gentleman's septic shock)

A 75-year-old male arrives by ambulance to the emergency room severely confused. His vitals are T 40 C, HR 120 bpm, BP 80/55 mmHg, RR 25. His wife explains that he injured himself about a week ago while cooking, and several days later his finger became infected, oozing with pus. He ignored her warning to see a doctor and even refused after he developed fever, chills, and severe fatigue yesterday. After being seen by the emergency physician, he was given antibiotics and IV fluids. Following initial resuscitation with IV fluids, he remains hypotensive. The ED physicians place a central venous catheter and begin infusing norepinephrine. Which of the following receptors are activated by norepinephrine? A. Alpha 1 B. Alpha 2 C. Alpha 1, Alpha 2, Beta 1 D. Alpha 1, Alpha 2, Beta 1, Beta 2 E. Alpha 1, Beta 1, Dopamine 1

A (flutamide is an anti-androgen that acts as a competitive inhibitor on testosterone receptors - used in combination with a GnRH agonist like leuprolide, it can treat advanced prostate cancer) (B; this is an aromatase inhibitor that treats estrogen-receptor positive breast cancer) (C; this is fertility medicine) (D; this a selective estrogen receptor modulator, used to treat estrogen-receptor positive medicine) (E; this is an antiandrogen, same as flutamide - this wouldn't be as good as complement therapy)

A 75-year-old male is diagnosed with advanced metastatic prostate cancer. After further evaluation and staging, the patient is started on flutamide therapy. Addition of which of the following medications to this patient's medication regimen would be of greatest benefit in the treatment of this patient's condition? A. Leuprolide B. Anastrozole C. Clomiphene D. Tamoxifen E. Cyproterone

D (this is preeclampsia - note gestational age, hypertension, and proteinuria - IV magnesium sulfate is used to prevent progression to eclampsia)

A female at 33 weeks gestation presents to her obstetrician with a complaint of occasional severe headaches and abdominal pain. Physical exam shows a blood pressure of 165/95 mmHg, and moderate facial and lower extremity edema. Urinalysis reveal 4+ albuminuria. The immediate drug treatment for this woman's condition is A. Calcium gluconate B. Carbamazepine C. Diazepam D. Magnesium sulfate E. Oxytocin

C (potency is defined as the ability of drug to exude effect at a particular concentration, in this case the EC50- the lower the concentration, the higher the potency - efficacy is defined as the maximum effect a drug can produce) (all four drugs have an equal efficacy, but A is the most potent because it can achieve this efficacy at the lowest concentration)

A laboratory is conducting a study to assess the safety, efficacy, and potency of a group of drugs before allowing the agents to proceed to clinical trials. Shown are the dose-response curves for four different drugs from the same class of medications (Drugs A, B, C, & D). Which of the following is true regarding these medications? A. Drug A has greater efficacy than Drug D B. Drug D has greater efficacy than Drug A C. Drug A has greater potency than Drug D D. Drug D has greater potency than Drug A E. All four drugs have equal potencies

B (600 --> 300 --> 150 --> 75 --> 37.5, this sequence shows the progress of four half lives - if four half lives occurred over two hours, then the half life must be half an hour, or 30 minutes) (this method only works for drugs with first-order kinetics, where the half life is constant - in zero order kinetics, the drugs have a constant rate of elimination, meaning the half life changes)

A patient is given 600 mg of a drug. The elimination process follows first-order kinetics. Two hours later, 37.5 mg remains in his body. What is the half-life of the drug? A. 15 minutes B. 30 minutes C. 40 minutes D. 60 minutes E. 90 minutes

E (the sky-high blood pressure and pounding headaches with flushing without pattern or replicability suggests pheochromocytoma, for which the repair is surgical - pheochromocytoma secretes stupid amounts of norepinephrine sporadically, causing all of the listed effects by acting on alpha receptors - an approved treatment for this is phenoxybenzamine, an irreversible inhibitor of alpha receptors) (atropine is anticholinergic, isoproterenol is a beta agonist, propranolol is a beta blocker, and phentolamine is reversible/competitive for alphas, which makes it a worse treatment option)

A patient presents with periods of severe headaches and flushing however every time they have come to the physician they have not experienced any symptoms. The only abnormal finding is a blood pressure of 175/100 mmHg. It is determined that the optimal treatment for this patient is surgical. Prior to surgery which of the following noncompetitive inhibitors should be administered? A. Atropine B. Isoproterenol C. Propranolol D. Phentolamine E. Phenoxybenzamine

C (octreotide is a somatostatin analogue, which decreases secretion of GH, and therefore decreases IGF-1 and VIP further downstream [this is why it is used to treat VIPoma] - ultimately this will cause a decrease in somatostatin as well due to feedback inhibition)

A patient presents with persistent diarrhea. Further examination reveals that the cause of the patient's diarrhea is a VIPoma. The patient is perscribed octreotide and blood work is conducted two weeks later. Which of the following would be observed in the patient's blood work? A. Increased somatostatin B. Increased IGF-1 C. Decreased GH D. Increased VIP E. Increased potassium

C (remember that bioavailability for an IV drug is always 100% - injecting 50 mg of compound X gives an AUC of 60 mghr/L, so injecting 100 mg of compound X should give an AUC of 120 - the oral form, administered at 100 mg, gave an AUC of 80, which means that, for the same administered amount, the oral form's bioavailability relative to the injected form's is 80/120, or 2/3, which equates to 66.6%) (the formula is absolute bioavailability = [oral AUC x IV dose]/[IV AUC x oral dose])

A pharmaceutical company is attempting to determine the bioavailability of a new glucose-lowering agent, Compound X, they have developed. They have previously studied the pharmacokinetics of both IV and oral forms of this drug. The company has obtained the following information regarding the IV formulation of Compound X: Administration of 50 mg of Compound X by IV yields an area under the curve (AUC) on a plasma drug concentration versus time plot of 60 mg hr/L. Additionally, the company has also obtained the following information regarding the oral formulation of Compound X: Administration of 100 mg of Compound X by a PO route yields an AUC on a plasma drug concentration versus time plot of 80 mg hr/L. Which of the following is the absolute bioavailability of Compound X? A. 58% B. 64% C. 67% D. 70% E. 74%

A (drug A shows a higher potency, but also a lower efficacy, than drugs B and C - because its maximum response is partial compared to the other drugs, it is a partial agonist)

A study is being conducted to test the potency and efficacy of three experimental drugs that act on the same receptor. The dose-response curves for these three drugs are shown. The most accurate statement concerning these results is A. Drug A is a partial agonist B. Drug A will likely b the most clinically useful C. Drug C has greater efficacy than drug B D. Drug C is more potent than drug B E. Drugs B and C have the same potency

D (misoprostol is a PGE1 analogue, a prostaglandin) (A; this is a main effect of misoprostol) (B; misoprostol is approved for the use of couneracting ulcerogenic effects of NSAIDs, which are caused by a decrease in gastric PGE2) (C; these are common side effects of misoprostol) (E; the uterine muscle is contracted by PGE2)

All of the following statements about misoprostol are true except A. It inhibits acid secretion by parietal cells B. When given to patients taking NSAIDs, it tends to replace endogenous PGE2 C. It causes a significant incidence of abdominal pain and diarrhea D. It is more effective than famotidine in healing gastric ulcers E. It can induce contraction of the uterus

D (the diagnosis is bipolar disorder, currently in a manic phase - the best treatment is a mood stabilizer, for which the first drug of choice is lithium - other first line drugs include valproate, quietiapine, or lamotrigine)

An 18-year-old college freshman is brought to the clinic by her parents. For the past two weeks, she has been sleeping two hours per night, has had difficulty concentrating, has racing thoughts, and rapid speech. Physical exam reveals osteopathic restrictions within the OA and suboccipital regions, and a cranial rhythmic impulse which is decreased in amplitude and frequency. Her mother insists that the patient is normally quite shy and has never acted like this before. The most appropriate pharmacotherapy to initiate is A. Carbamazepine B. Fluoxetine C. Haloperidol D. Lithium carbonate E. Lorazepam

A (the patient likely overdosed on amitriptyline, a tricyclic antidepressant, which causes toxic features such as seizure and torsades de pointes - the toxic effects of tricyclic antidepressants can be remembered as the three Cs - coma, convulsion, and cardiotoxicity) (overdose of the other drugs would have features of serotonin syndrome, such as rigidity, hyperreflexia, and hallucinations)

An 18-year-old female with a history of major depression is rushed to the hospital by emergency medical services after being found on the floor of her bedroom, obtunded. Her father notes that the patient has been treated with an antidepressant for years, although he is unsure what medication she was currently taking. As the physician is speaking with the patient's father, the patient begins to have convulsions and ultimately passes away. Review of premorbid EKG shows torsades de pointes. Overdose with which of the following medications is the most likely cause of this patient's condition? A. Amitriptyline B. Sertraline C. Bupropion D. Venlafaxine E. Fluoxetine

B (the stem describes a patient who has grand mal [tonic clonic] and petit mal [absence] seizures - valproic acid is the best drug for treating both simultaneously) (A; good for complex and partial seizures, but not first line for tonic-clonic and no effect on absence) (C; first line for tonic clonic, no effect on absence) (D; first line for absence, no effect on tonic clonic) (E; first line for acute status epilepticus)

An 8-year-old boy is being seen in your neurology clinic for grand mal seizures. While speaking with the child, you notice that he frequently asks you to repeat yourself, and looks at you occasionally with a blank stare. Which of the following medications would be most appropriate for this patient? A. Gabapentin B. Valproic acid C. Phenytoin D. Ethosuximide E. Lorazepam

D (the diagnosis is Tourette's, which is treated preferentially with second generation antipsychotic drugs, which are agonists at the D2 receptor)

An 8-year-old boy is brought to the clinic by his mother. She is concerned that he often clears his throat repeatedly. He also shakes his head from side-to-side and has repetitive blinking motions. The tics have interfered with his school productivity, and he is frequently made fun of by other classmates because of the tics. On examination you note rapid and erratic cranial motion with a left lateral torsion strain pattern. What class of medication is used for treatment of this condition? A. Beta blocker B. Anticholinesterase inhibitor C. Dopamine and norepinephrine reuptake inhibitor D. D2 receptor agonist E. Monoamine oxidase inhibitor

B (methylphenidate [Ritalin] is the first line stimulant drug for the treatment of ADHD, which acts by inhibiting reuptake of dopamine and norepinephrine) (A; SSRIs are first line drugs for several disorders, but ADHD is not one of them) (C; these are first line drugs for myasthenia gravis and some forms of dementia) (D; D2 receptors are often blocked as a mode of action for antipsychotic medication) (E; MAOIs are not first line treatment for anything, due to severe side effects, but it can be used for anxiety and depressive disorders)

An 8-year-old boy presents with his parents for psychiatric evaluation. His parents report that he has been having difficulties following directions at home. When he is reminded of his chores, he starts them but then stops after some time. When they confront him, he becomes irritated and frustrated with his parents. He has not been completing his class work and the teachers recommend he be evaluated by a psychiatrist. During your examination, the child starts to answer questions and initially cooperates, but later becomes distracted. The child was prescribed a first-line stimulant medication. What is the mechanism of action of this drug? A. 5-HT reuptake inhibitor B. Blocks reuptake of dopamine and norepinephrine C. Cholinesterase inhibitor D. D2 receptor agonist E. Monoamine oxidase inhibitor

A (ultimately, this prevents degranulation of the mast cells to prevent asthma attacks) (B; steroids act on nuclear receptors) (C; this would be the -lukast class of drugs) (D; this is theophylline) (E; this is irrelevant to the patient, since asthma is a type I hypersensitivity reaction)

An 8-year-old female is given omalizumab for the treatment of bronchial asthma. Omalizumab treats asthma through which mechanism? A. Inhibition of IgE binding to mast cells B. Binding to nuclear receptors C. Inhibition of leukotriene binding to receptor D. Inhibition of phosphodiesterase breakdown of cAMP E. Mediating type IV hypersensitivity reaction

D (the patient is being treated for his heart problems with digoxin, for which nausea, vomiting, and yellow vision are toxic side effects - why is he experiencing toxic side effects? - one possible reason is that digoxin is renally excreted, and renal insufficiency would increase digoxin's concentration) (A, E; these would not increase digoxin toxicity) (B; hypokalemia, not hyperkalemia, increases digoxin toxicity) (C; digoxin is metabolized in the kidneys, not the liver) (remember the mechanism of action of digoxin - blockade of Na-K-ATPases on cardiac myocytes, and increased vagal stimulation)

An 81-year-old man being treated for congestive heart failure and atrial fibrillation develops nausea, vomiting, and blurry yellow vision. Which of the following pathologic states would best explain this patient's presentation? A. Hypernatremia B. Hyperkalemia C. Hepatic insufficiency D. Renal insufficiency E. Metabolic acidosis

B (basically, NSAID's reduce the efficacy of loop diuretics like furosemide - in addition to their diuretic effect, loop diuretics increase renal blood flow via prostaglandin release - if a patient takes an NSAID like naproxen [in this case, for his arthritis], this antagonizes the prostaglandin release, reduces renal blood flow, and therefore reduces the diuresis, leading to fluid retention and exacerbation of conditions like pulmonary edema and congestive heart failure)

An 82-year-old male with osteoarthritis, hypertension and allergic rhinitis was diagnosed 3 months ago with congestive heart failure and resultant pulmonary edema. Until recently, the patient's symptoms had been well controlled by furosemide therapy. Since beginning a new medication for a comorbid condition, he is more short of breath. A chest radiograph of the patient is shown in Image A. Which of the following medications is likely responsible for these worsening symptoms? A. Spironolactone B. Naproxen C. Zileuton D. Montelukast E. Hydrochlorothiazide

D (this is a rule of thumb - it takes about 4 half lives to reach a steady-state concentration of over 90%, and 1.5 x 4 = 6)

An experimental infusable drug, X729, is currently being studied to determine its pharmacokinetics. The drug was found to have a half life of 1.5 hours and is eliminated by first order kinetics. What is the minimum number of hours required to reach a steady state concentration of >90%? A. 1.5 B. 3 C. 4.5 D. 6 E. 7.5

B (A; antacids are as good as H2 blockers at healing ulcers) (C; antacids neutralize secreted acid - they do not inhibit acid secretion like PPIs do) (D; as the dosage is raised, the laxative effect of magnesium predominates) (E; antacids do not have bactericidal effects on H pylori)

Antacids that are used in the treatment of duodenal ulcers A. are less efficacious than ranitidine in promoting healing of ulcers B. are usually mixtures of aluminum hydroxide and magnesium hydroxide C. are effective in part because they suppress gastric acid secretion D. are more apt to cause constipation as the dosage is raised E. have important bacteriocidal effects on H pylori

C (the diagnosis is Legionaires disease, caused by Legionella pneumophila - the most appropriate treatments are floroquinolones and macrolides - when using a floroquinolone in elderley patients, make sure to monitor for side effects of hepatotoxicity and tendon damage)

As part of a major outbreak of a syndrome that includes fever and mild respiratory symptoms, a 72-year-old female presents to the emergency department with painful breathing and wheezing. Physical examination reveals a temperature of 103.1 F and rales are heard over both lungs upon auscultation. Epidemiological research reveals a connection to a local amusement park, where the patient used a mist machine to cool off from the heat of the day. A scant sputum sample is collected and reveals growth on buffered charcoal yeast extract agar. What is the most appropriate antibiotic for treatment of this patient's infection? A. Ceftriaxone B. Isoniazid C. Levofloxacin D. Penicillin G E. Trimethoprim-sulfamethoxazole

A (neostigmine is an indirect cholinomimetic agent that doesn't penetrate the CNS [so no miosis] used to reverse non-depolarizing muscle relaxants at the end of surgery - it causes bradycardia if not also given with a muscarinic antagonist such as glycopyrrolate)

As the surgeon is dressing the incisions upon completion of an elective cholecystectomy, the anesthesiologist administers neostigmine immediately followed by glycopyrrolate. If neostigmine were given without glycopyrrolate, what changes would you expect to see in the patient? A. Reversal of neuromuscular blockade, bradycardia B. Reversal of neuromuscular blockade, tachycardia C. Miosis, bradycarida D. Miosis, tachcardia E. Reversal of neuromuscular blockade, miosis

C (the concern here, as in all nuclear/radioactive accidents, is radioactive iodide, which can be taken up by the thyroid gland and cause thyroid cancer- prophylaxis would involve a competitive iodide compound to decrease the uptake of the radioactive form) (A, B; these can treat methemoglobinemia) (D, E; these can treat lead poisoning)

At 10 a.m. this morning, a semi-truck carrying radioactive waste toppled over due to a blown tire. One container was damaged, and a small amount of its contents leaked into the nearby river. You are a physician on the government's hazardous waste committee and must work to alleviate the town's worries and minimize the health hazards due to the radioactive leak. You decide to prescribe a prophylactic agent to minimize any retention of radioactive substances in the body. Which of the following do you prescribe? A. Methylene blue B. Vitamin C C. Potassium iodide D. EDTA E. Succimer

B (these are the therapeutic indications for these drugs) (A; bismuth is part of the triple therapy for H pylori, but sucralfate is not) (C, D; neither drug is an antacid or an antihistamine) (E; sucralfate contains aluminum but bismuth does not)

Both sucralfate and bismuth subcitrate A. eradicate H pyloric infections in the stomach B. form a protective barrier on ulcer craters C. have significant acid neutralizing capability D. have significant H2 blocking actions E. can produce aluminum toxicity in renally impaired patients

D (ranitidine is the improved form of the prototype drug, cimetidine, which has the same selectivity and efficacy, but also has a nasty side-effect profile of antiandrogenic effects, inhibition of the P450 system, and CNS effects)

Compared to ranitidine, cimetidine A. is more effective in healing duodenal ulcers B. is less selective for H2 receptors C. produces a lower rate of recurrence of ulcers after cessation of treatment D. is a more potent inhibitor of the hepatic P450 system E. has little tendency to produce antiandrogenic effects

C (ipratropium is an anti-cholinergic that reduces smooth muscle contraction in the airway, aka bronchospasm - ipratropium is the drug of choice for COPD)

In patients with chronic obstructive pulmonary disease, stimulation of muscarinic acetylcholine receptors results in an increase in mucus secretion, smooth muscle contraction and bronchoconstriction. The end result is an increase in airway resistance. Which of the following pharmacologic agents interferes directly with this pathway? A. Epinephrine B. Theophylline C. Ipratropium D. Albuterol E. Metoprolol

B (cilostazol is a phosphodisterase 3 inhibitor, meaning it reduces the degradation of intracellular cAMP - in cardiomyocytes, increased cAMP means increased force of contraction)

The drug cilostazol is known for its ability to relax vascular smooth muscle and therefore cause vasodilation through its inhibition of phosphodiesterase 3. Given this mechanism of action, what other effect would be expected? A. Increased left ventricular end-diastolic volume B. Positive inotropy C. Negative chronotropy D. Angioedema E. Antiarrhythmic action

C (I mean, kind of a rote recall thing here)

Which of the following best describes diphenoxylate? A. Organic stool softener; promotes colonic motility B. Poorly absorbed; used for rapid effect before bowel surgery C. Poorly absorbed opioid; slows transit of intestinal contents D. Promotes healing of peptic ulcers E. Promotes secretion of water into the colon; chronic use can produce "cathartic colon"

A (drugs B-E are H1 receptor blockers - gastric acid secretion is stimulated by H2 receptors and agonists and PGI2 inhibits this)

Which of the following inhibits histamine-induced gastric acid secretion? A. PGI2 B. Pyrilamine C. Diphenhydramine D. Meclizine E. Astemizole

E (A; ZE is caused by a gastrin secreting tumor, leading to high acid content in the stomach - omeprazole, a PPI, effectively treats this) (B; the PPI effects is irreversible and lasts for up to 24 hours, while omeprazole's half life is only 1 hour) (C; omeprazole is a prodrug activated by low pH) (D; omeprazole is more effective than H2 blockers because it is longer lasting and causes greater inhibition)

Which of the following statements about omeprazole is the least valid? A. It is indicated for the therapy of Zollinger-Ellison syndrome B. It has a duration of action that far outlasts its lifetime in the plasma C. Its formation of active drug requires a low pH D. It causes greater inhibition of gastric acid secretion than does ranitidine E. It heals ulcers more slowly than does cimetidine

D (massive hepatic necrosis is a lethal side effect of the inhaled anesthetic, halothane)

While on a teaching sabbatical in Uruguay, a pathologist examined the excised liver of an 18-year-old otherwise healthy female who passed away due to massive hepatic necrosis 5 days after she underwent general anesthesia to repair a fractured femur. Which of the following is a general anesthetic most likely responsible for her death? A. Bupivacaine B. Lidocaine C. Midazolam D. Halothane E. Desflurane

D (this patient is a chronic opioid abuser - the X ray shows the constipation associated with opioid abuse, another side effect of which is miosis) (A; this is a side effect of acute opioid use - however, opioid users develop tolerance to all the side effects, except two - miosis and constipation)

You are seeing a patient in clinic with chronic low back pain. Despite frequent physical therapy, and multiple attempts to control the pain with NSAIDs, the patient requires morphine to control his pain. He has been using morphine for several months now and the dose required to control his pain has doubled in that time. He has been complaining of persistent abdominal pain, and in the course of evaluating his abdominal pain you obtain an abdominal radiograph. Which additional opioid side effect would you expect to see in this patient? A. Low respiratory rate B. Rhinorrhea C. Sweating D. Miosis E. Diarrhea

C (the drug in question is an aminoglycoside)

You are treating a neonate with meningitis using ampicillin and a second antibiotic, X, that is known to cause ototoxicity. What is the mechanism of antibiotic X? A. It binds the 50S ribosomal subunit and inhibits peptidyltransferase B. It binds the 50S ribosomal subunit and inhibits formation of the initiation complex C. It binds the 30s ribosomal subunit and inhibits formation of the initiation complex D. It binds the 30s ribosomal subunit and reversibly inhibits translocation E. It binds the 50s ribosomal subunit and reversibly inhibits translocation


Kaugnay na mga set ng pag-aaral

Encumbrances & Ownership-Multi choice questions

View Set

AP Biology The Complete Edition :)

View Set

Pharmacology- ch12 Parenteral administration (ID, IM,subcut, IV)

View Set

Chapter 20: Documenting and Reporting

View Set

Chapter 3.B - Legal Issues: Liens & Easements

View Set

Experiment 10 - Aldehydes and Ketone

View Set